ML070610451

From kanterella
Jump to navigation Jump to search
Draft - RO & SRO Written Exam (Folder 2)
ML070610451
Person / Time
Site: Pilgrim
Issue date: 12/11/2006
From:
NRC Region 1
To:
Entergy Nuclear Operations
Sykes, Marvin D.
Shared Package
ML060800104 List:
References
Download: ML070610451 (199)


Text

ES-401 Sample Written Examination Form ES-401-5 Question Worksheet

-4 Examination Outline Cross-reference: Level RO SRO Tier # 2 Group # 1 KIA # 203000 - RHWLPCI:

fnjection Mode - K2.03 Importance Rating 2.7 Knowledge of Electrical power supplies to the following: Initiation Logic Proposed Question: Common I With the plant at 100% power a loss of 125 VDC Bus D16 occurs. While in this condition, a large recirc loop B pipe break results in a reactor scram and rapid vessel depressurization.

Given the loss of I 2 5 VDC and its impact on the LPCl Loop Select Logic:

LPCI loop select logic will A. Select the Aloop for injection B. Select the B loop for injection C. Not select either loop for injection

~d D. Will open both loop A and 6 injection valves Proposed Answer: A Y-Explanation (Optional):

A. Correct response. Loop select logic is supplied from both D16 (via D4) and D5 and will still function as designed.

B. Incorrect. Loop select logic is supplied from both D16 (via D4)and D5 and will still function as designed. Since the Bloop is broken, Aloop is chosen for injection.

C Incorrect. Loop select logic is supplied from both D16 (via 04) and D5 and will still function as designed. Since the B loop is broken, A loop is chosen for injection.

D. Incorrect. Loop select logic is supplied from both D16 (via D4) and D5 and will still function as designed. Since the 6 loop is broken, A loop is chosen for injection.

Technical Reference(s) PNPS 5.3.1 I,page 7, (Attach if not previously provided) paragraph [8]

Proposed references to be provided to applicants during examination: NONE Learning Objective: (As available)

L, Page 1 of 203

ES-401 Sample Written Examination Form ES-401-5 Question Worksheet Question Source: Bank ##

LJ Modified Bank # (Note changes or attach parent)

New X Question History: Last NRC Exam Question Cognitive Level: Memory or Fundamental Knowledge Comprehension or Analysis -x 10 CFR Part 55 Content: 55.41 (7) 55.43 Comments:

Page 2 of 203

ES-401 Sample Written Examination Form ES-401-5 Question Worksheet

.d Examination Outline Cross-reference: Level RO SRO Tier # 2 Group # 2 WA # 205000 K3.03 Importance Rating 3.8 Knowledge of the effect of a loss or malfunction of the Shutdown cooling system will have on the following: Reactor Temperatures.

Proposed Question: Common 2 The plant is operating with RHR in shutdown cooling during the early stages of a reactor shutdown for refueling.

If the operators-(I ) , reactor vessel temperatures would- (2)-?

A. (I) Removed the Fuel Pool Cooling system from service.

(2) Increase.

B. (I) Observed that reactor pressure had increased to 60 psig.

(2) Decrease.

Ll C. (I) Opened the inservice RHR Heat Exchanger Bypass Valve.

(2) Decrease.

D. (I) Reported a loss of the Salt Service Water System.

(2) Increase.

Proposed Answer: D Explanation (Optional):

A. Incorrect. FPCC has no direct tie to the vessel in the given condition.

B. Incorrect. SDC isolates at 70 psig, Increase in pressure will cause temps to increase.

C. Incorrect. Opening the valve provides less flow to heat exchanger. Temps increase.

D. Correct. Salt Service Water cools RBCCW which cools RHR. Loss of heat sink will increase temps. Essentially a loss of SDC.

Technical Reference(s) RHR Reference text, Rev. 7, (Attach if not previously provided) page 67 section G.13 Page 3 of 203

ES-401 Sample Written Examination Form ES-401-5 Question Worksheet Proposed references to be provided to applicants during examination: NONE Learning Objective: (As available)

Question Source: Bank #

Modified Bank # (Note changes or attach parent)

New X Question History: Last NRC Exam Question Cognitive Level: Memory or Fundamental Knowledge Comprehension or Analysis X 10 CFR Part 55 Content: 55.41 (7) 55.43 Comments:

Page 4 of 203

ES-401 Sample Written Examination Form ES-401-5 Question Worksheet L.J Examination Outline Cross-reference: Level RO SRO Tier # 2 Group # 1 WA # 206000 A4.01 Importance Rating 3.8 Ability to manually operate and/or monitor in the control room: Turbine speed controls.

Proposed Question: Common 3 HPCl is in pressure control during a reactor shutdown and cooldown.

To maintain a constant flowrate with HPCl you must direct the operators to in accordance with PNPS 2.2.21 5,HPCI Injection and pressure control.

A. Place controller FIC 2340 Injection Flow Control in BAL and manually adjust flow with the potentiometer as reactor pressure decreases.

B. Place controller FIC 2340 Injection Flow Control in BAL. ONLY HPCl speed will be automatically maintained as reactor pressure decreases.

..& C. Maintain controller FIC 2340 Injection Flow Control in AUTO. HPCl speed will vary and flowrate will be maintained as reactor pressure decreases.

D. Maintain controller FIC 2340 Injection Flow Control in MANUAL which will maintain HPCl flow constant based on reactor pressure.

Proposed Answer: C Explanation (Optional):

A. Incorrect. Balance is only used to match auto and manual before a swapover.

6. Incorrect. Balance is only used to match auto and manual before a swapover.

C. Correct. In AUTO or BAL, flow demand signal, as determined by the FIC set point tape, controls turbine speed. In MAN, the operator controls the output signal, by adjusting its manual control potentiometer. With the controller in AUTO (system operatinq to maintain the selected flowrate iniectinq to the RPV), a reduction in reactor pressure will result in HPCls control valve closing down to maintain the flow rate constant. With the controller in MANUAL (system operating to maintain the selected turbine speed and injecting to the RPV), a reduction in reactor pressure will require an operator to reduce the speed potentiometer set point to maintain a constant flow rate.

D. Incorrect. Manual controls HPCl speed based on controller output, not reactor pressure.

Page 5 of 203

ES-401 Sample Written Examination Form ES-401-5 Question Worksheet v Technical Reference(s) HPCl Ref. Text. ,Rev.8, section (Attach if not previously provided)

B.7, page 23. & Section D.5.

Proposed references to be provided to applicants during examination: NONE Learning Objective: (As available)

Question Source: Bank #

Modified Bank # (Note changes or attach parent)

New X Question History: Last NRC Exam Question Cognitive Level: Memory or Fundamental Knowledge Comprehension or Analysis X 10 CFR Part 55 Content: 55.41 (7) 55.43 Comments:

J Page 6 of 203

ES-401 Sample Written Examination Form ES-401-5 Question Worksheet

.-/

Examination Outline Cross-reference: Level RO SRO Tier # 2 Group # I WA # 206000 K2.01 Importance Rating 3.2 Knowledge of electrical power supplies to the following: system valves.

Proposed Question: Common 4 Which ONE of the following HPCl components would be affected by a loss of Bus D-8?

A. HPCl Turbine Gland Seal Vacuum Pump.

B. HPCl Turbine Aux Oil Pump.

C. HPCl Pump Suction from CST M0-2301-6.

D. HPCl Pump Discharge Valve MO-2301-9.

-4' Proposed Answer: C Explanation (Optional):

A. Incorrect- powered from bus D-9 (250VDC)

6. Incorrect - powered from bus D-9 (250 VDC)

C. Correct - powered from 125 VDC D-8 D. Incorrect - powered from bus D-9(250VDC)

Technical Reference(s) HPCl reference text, Rev. 8 , (Attach if not previously provided)

Section E . l I.f.& g.

Proposed references to be provided to applicants during examination: NONE Learning Objective: (As available)

Question Source: Bank #

Modified Bank # (Note changes or attach parent)

New X Question History: Last NRC Exam Question Cognitive Level: Memory or Fundamental Knowledge X

'v' Page 7 of 203

ES-401 Sample Written Examination Form ES-401-5 Question Worksheet Comprehension or Analysis 10 CFR Part 55 Content: 55.41 (7) 55.43 Comments:

Page 8 of 203

ES-401 Sample Written Examination Form ES-401-5 Question Worksheet

-4 Examination Outline Cross-reference: Level RO SRO Tier # 2 Group # I WA # 209001 K1.08 Importance Rating 3.2 Knowledge of the physical andlor cause effect relationships between core spray and the following: AC electrical power.

Proposed Question: Common 5 The plant is starting up when a loss of offsite power occurs. Power was restored with the EDGs with the exception of Power Center B-17. A leak then develops in the drywell causing a change in containment parameters. The current plant conditions are:

Reactor is shutdown, the initial scram has not been reset.

Reactor Level is 15 inches.

Reactor Pressure is 480 psig.

Drywell Temperature 140 degrees.

Drywell Pressure is 2.8 psig.

Core Spray inboard injection valve MO-1400-25A is -(I) and valve MO-1400-258 is -(2)-..

-e B. (1) Closed (2) Closed D. ( I ) Open (2) Closed Page 9 of 203

ES-401 Sample Written Examination Form ES-401-5 Question Worksheet Proposed Answer: B

~d Explanation (Optional):

A. Incorrect. Power not available to open MO-1400-25A.

8. Correct. MO-1400-25A is closed due to power not available and valve MO-1400-25B is closed because Rx pressure is >400 psig.

C. Incorrect. MO-1400-258 is closed because Rx pressure is >400 psig.

D. Incorrect. Power not available to open MO-1400-25A.

Technical Reference(s) CS Reference text, Rev.4, (Attach if not previously provided) page 25, section F.l .b.

Proposed references to be provided to applicants during examination: NONE Learning Objective: (As available)

Question Source: Bank #

Modified Bank # (Note changes or attach parent)

New X Question History: Last NRC Exam

c. Question Cognitive Level: Memory or Fundamental Knowledge Comprehension or Analysis X 10 CFR Part 55 Content: 55.41 (7) 55.43 Comments:

Page 10 of 203

ES-401 Sample Written Examination Form ES-401-5 Question Worksheet L l Examination Outline Cross-reference: Level RO SRO Tier # 2 2 Group # 1 1 WA # 21I000 K1.02 Importance Rating 2.7 Knowledge of the physical relationships and/or cause effect relationships between Standby Liquid Control System and the following:

Core plate differential pressure indication.

Proposed Question: Common 6 The Standby Liquid Control sparger line is used for which of the following purposes?

I. Core plate differential pressure measurement.

II. Core Spray system line break detection.

Ill. Jet Pump differential pressure measurement.

A. I , I I and Ill.

B. I and I I only.

C. I I and Ill only.

u D. I and Ill only.

Proposed Answer: A Explanation (Optional): Question matches W A because it addresses the physical relationship between SLC and Core Plate DP.

A. Correct. The injection sparger is a multiple purpose line, which is used for core plate differential pressure measurement, core spray system line break detection, jet pump differential pressure measurement, and boron injection. If a break in the line occurs core plate delta P would be 0 and indicate downscale.

B. Incorrect. Also used for Ill.

C. Incorrect. Also used for I.

D. Incorrect. Also used for il.

Technical Reference@) SLC Reference Text, Rev. 3, (Attach if not previously provided)

Page 13, section E.9.

Proposed references to be provided to applicants during examination: NONE Page 11 of 203

ES-401 Sample Written Examination Form ES-401-5 Question Worksheet Learning Objective: (As available)

Question Source: Bank #

Modified Bank # (Note changes or attach parent)

New X Question History: Last NRC Exam Question Cognitive Level: Memory or Fundamental Knowledge X Comprehension or Analysis 10 CFR Part 55 Content: 55.41 (2) 55.43 Comments:

12/1 Gil - KA doesnt match 12/19 - question revised Page 12 of 203

ES-401 Sample Written Examination Form ES-401-5 Question Worksheet

+~-2 Examination Outline Cross-reference: Level RO SRO Tier # 2 2 Group # I 1 KIA # 21 1000 A4.01 Importance Rating 3.9 Ability to manually operate and/or monitor in the control room: Tank Level.

Proposed Question: Common 7 A loss of Instrument Air occurs to the Standby Liquid Control (SLC) tank bubbler dip tu be.

The effect on indicated SLC tank level will be:

A. Local and Control Room indication will fail high.

6. Local and Control Room indication will fail low.

C. Local indication will fail high. Control room indication will fail low.

.L/

D. Local indication will fail low. Control room indication will fail high.

Proposed Answer: B Explanation (Optional):

A. Incorrect. Indications fail low.

B. Correct. Instrument air supplies the air for the bubbler dip tube, which is associated with the storage tank level transmitters. Loss of instrument air will cause the local and control room level indicators to fail low.

C. Incorrect. Both indications fail low.

D. Incorrect. Both indications fail low.

Technical Reference(s) SLC Reference Text Rev.3 (Attach if not previously provided)

Page 16. Section E. 1.

Proposed references to be provided to applicants during examination: NONE Learning Objective: (As available)

-/

Question Source: Bank #

Page I 3 of 203

ES-401 Sample Written Examination Form ES-401-5 Question Worksheet Modified Bank # (Note changes or attach parent)

New X Question History: Last NRC Exam Question Cognitive Level: Memory or Fundamental Knowledge X Comprehension or Analysis 10 CFR Part 55 Content: 55.41 (2) 55.43 Comments:

12/1 - Gil - none Page 14 of 203

ES-401 Sample Written Examination Form ES-401-5 Question Worksheet

~4 Examination Outline Cross-reference: Level RO SRO Tier # 2 Group # 1 WA # 212000 RPS - K4.08 Importance Rating 4.2 Knowledge of the R actor Protection System design features and or interlocks which rovide for the following: Complete control rod insertion following Scram signal generation.

Proposed Question: Common 8 The Reactor Protection System (RPS) has just received a valid trip signal on both channels A and B (Reactor Water Level Low).

Which ONE (I) of the following correctly describes how RPS functions to insert control rods?

A. Both sets of scram pilot valve solenoids are de-energized, allowing instrument air to be vented from the scram valves. This permits the scram valves to open, causing all control rods to be rapidly inserted (scrammed) by HCU accumulator pressure.

d B. Both sets of scram pilot valve solenoids are energized allowing instrument air to be vented from the scram pilot air header. This permits the scram valves to open, causing all control rods to be rapidly inserted (scrammed) by HCU accumulator pressure C. Both backup scram valve solenoids de-energize, which then vents instrument air pressure via the backup scram valves from the scram pilot valve air header, causing all control rods to be rapidly inserted (scrammed) by HCU accumulator pressure.

D. Both backup scram valve solenoids energize, which then vents instrument air pressure via the scram pilot valves directly to atmosphere, causing all control rods to be rapidly inserted (scrammed) by HCU accumulator pressure.

Page 15 of 203

ES-401 Sample Written Examination Form ES-401-5 Question Worksheet Proposed Answer: A L/ Explanation (Optional):

A. Correct. Scram pilot valve solenoids are de-energized, allowing instrument air to be vented from the scram valves. This permits the scram valves to open, causing all control rods to be rapidly inserted (scrammed) by HCU accumulator pressure.

6. Incorrect. Scram pilot valve solenoids are de-energized, not energized.

C. Incorrect. Both backup scram valve solenoids energized, not de-energized.

D. Incorrect. Air is vented from the scram pilot air header via the backup scram valves.

Technical Reference(s) RPS reference text, Rev.4, (Attach if not previously provided) page 11 (step C.6.), page 31 (step F.3.a.)

Proposed references to be provided to applicants during examination: NONE Learning Objective: (As available)

Question Source: Bank #

Modified Bank # (Note changes or attach parent)

New X Question History: Last NRC Exam Question Cognitive Level: Memory or Fundamental Knowledge Comprehension or Analysis X 10 CFR Part 55 Content: 55.41 (7) 55.43 Comments:

Page 16 of 203

ES-401 Sample Written Examination Form ES-401-5 Question Worksheet

'%d Examination Outline Cross-reference: Level RO SRO Tier # 2 Group # 1 WA # 212000 K5.02 Importance Rating 3.3 Knowledge of the operational implications of the following concepts as they apply to RPS: Specific Logic Arrangements.

Proposed Question: Common 9 The scram discharge volume high water level bypass switch was placed in bypass to reset a scram and was left in that position.

During the subsequent startup with the mode switch in Startup and power at Range 2 on the IRMs the following alarms are received.

SDlV WEST NOT DRAINED/ Panel C905R G I SDIV LEVEL HIGH/Panel C905R D3 PRIOR to the alarms the Scram Discharge Volume vent and drain valves were ,(I)-?

AFTER the alarms the Scram Discharge Volume vent and drain valves were -(2) -?

B. (1)Open (2) Closed D. (1) Closed (2) Closed Page 17 of 203

ES-401 Sample Written Examination Form ES-401-5 Question Worksheet Proposed Answer: B

'LJ Explanation (Optional):

A. Incorrect. The scram is not bypassed with mode switch in startup.

B. Correct. Being in Startup requires a mode switch change. Changing the mode switch position deactivates the bypass switch. The SDV scram is now active and the valves close on the alarm.

C. Incorrect. The scram is not bypassed with mode switch in startup.

D. Incorrect. When the scram was initially reset the valves were open.

Technical Reference(s) RPS Reference Text, Rev. 4, (Attach if not previously provided) page 27, section D.5.

Proposed references to be provided to applicants during examination: NONE Learning 0bjective: (As available)

Question Source: Bank #

Modified Bank # (Note changes or attach parent)

New X

'.-/'

Question History: Last NRC Exam Question Cognitive Level: Memory or Fundamental Knowledge Comprehension or Analysis X 10 CFR Part 55 Content: 55.41 (5) 55.43 Comments:

Page 18 of 203

ES-401 Sample Written Examination Form ES-401-5 Question Worksheet J Examination Outline Cross-reference: Level RO SRO Tier # 2 2 Group # 1 1 WA # 2 15003 K4.04 Importance Rating 2.9 Knowledge of Intermediate Range Monitor (IRM) System design features and/or interlocks which provide for the following: Varying system sensitivity levels using range switches.

Proposed Question: Common 10 The Intermediate Range Monitor (IRM) , in conjunction with the IRM range switches adjust the sensitivity of the IRM circuitry.

A. Voltage preamplifiers

8. Attenuators C. Mean square analog circuits D. Inverters 4 .

Proposed Answer: B Explanation (Optional):

A. Incorrect. Only the attenuators affect IRM sensitivity (see reference text).

8. Correct. The IRM attenuators, in conjunction with the IRM Range switches on C905, simply adjusts the sensitivity of the IRM circuitry, so that a single strip-chart on C905 can be used to display five full decades of reactor power. Consequently, the least attenuation occurs on Range I,and the most attenuation occurs on Range I O .

C. Incorrect. Only the attenuators affect IRM sensitivity (see reference text).

D. Incorrect. Only the attenuators affect IRM sensitivity (see reference text).

Technical Reference(s) IRM Reference Text, Rev. 5, (Attach if not previously provided)

Section C.3.

Proposed references to be provided to applicants during examination: NE Learning Objective: (As available) u Page 19 of 203

ES-401 Sample Written Examination Form ES-401-5 Question Worksheet Question Source: Bank #

\d Modified Bank # (Note changes or attach parent)

New X Question History: Last NRC Exam Question Cognitive Level: Memory or Fundamental Knowledge X Comprehension or Analysis 10 CFR Part 55 Content: 55.41 (7) 55.43 Comments:

12/1- Gil - needs more refs to verify C is not also correct Page 20 of 203

ES-401 Sample Written Examination Form ES-401-5 Question Worksheet Examination Outline Cross-reference: Level RO SRO Tier # 2 2 Group # 1 1 KIA # 215004 K2.01 Importance Rating 2.6 Knowledge of electrical power supplies to the following: SRM channels/ detectors.

Proposed Question: Common 11 If a loss of power occurred on 120 VAC panel Y - I , how would the SRMs be affected?

A. ONLY SRMs Aand C would have lost their battery charger power supply.

B. ONLY SRMs B and D would have lost their battery charger power supply.

C. No affect. The SRMs battery power is supplied by a 125 VDC battery charger D. All SRMs would have lost their battery charger power supplies.

u Proposed Answer: D Explanation (Optional):

A. Incorrect. They all would have lost their battery charger power supply.

B. Incorrect. Y-1 (not Y-2) supplies 120 VAC to all SRM chargers - see figure #I.

C. Incorrect. Y-I supplies 120 VAC to all SRM chargers - see figure # I .

D. Correct. Y-1 supplies 120 VAC to all SRM chargers for backup power - see figure #I.

Technical Reference(s) 24VDC Reference Text, Rev.4, (Attach if not previously provided)

Figure #I, 24 VDC Distribution Proposed references to be provided to applicants during examination: NONE Learning Objective: (As available)

Question Source: Bank #

Modified Bank # (Note changes or attach parent)

New X

  • i Question History: Last NRC Exam Page 21 of 203

ES-401 Sample Written Examination Form ES-401-5 Question Worksheet I d Question Cognitive Level: Memory or Fundamental Knowledge X Comprehension or Analysis 10 CFR Part 55 Content: 55.41 (7) 55.43 Comments:

12/1 - Gil - make question more operational 12/19 - Steve - revised Page 22 of 203

ES-401 Sample Written Examination Form ES-401-5 Question Worksheet Examination Outline Cross-reference: Level RO SRO Tier # 2 Group # 1 ~~~ ~~

WA # 215005 - A3.03 Importance Rating 3.3 Ability to monitor automatic operations of the Average Power Range Monitor/Local Power Range Monitor Systems including: Meters and recorders.

Proposed Question: Common 12 The Average Power Range Monitors (APRM) have a count circuit that ensures there are enough Local Power Range Monitors (LPRM) for proper APRM operation. During your control room walkdown prior to assuming the shift, you find that 4 LPRMs assigned to the B APRM are bypassed.

The plant is operating at 90% power. No other LPRMs are bypassed.

Which ONE (I) of the following correctly describes (1) the value of the B APRM count circuit AND (2) automatic actions which should have occurred, if any?

A. (I) 50%.

L../ (2) A rod block and a half scram should have occurred.

B. (1)40%.

(2) A rod block and a half scram should have occurred.

C. (1)50%.

(2) A half scram ONLY should have occurred D. (1)40%.

(2) No automatic actions should have occurred.

Page 23 of 203

ES-401 Sample Written Examination Form ES-401-5 Question Worksheet Proposed Answer: A 4

Explanation (Optional):

A. Correct. Each LPRM sends a fixed current signal to the count circuit when its mode switch is in OPERATE. Sixteen LPRMs are used by APRM channels A, C and E.

Fourteen LPRMs are used by APRM channels B, D and F. An inop trip is initiated when an APRM module is unplugged, less than 11 assigned LPRMs are operating, the APRM mode switch in panel 937 is not in OPERATE, or an FCTR card critical self test failure.

An inop trip generates both a scram signal and a rod block signal. In this case only 10 LPRMs are operating for the B APRM. Each one indicates 5% on the APRM meter (50%

total) when the switch is taken to count.

B. Incorrect. Not 40%.

C. Incorrect. Also generates rod block.

D. Incorrect. Not 40% Generates a rod block and half scram.

Technical Reference(s) APRM reference text, Rev.6, (Attach if not previously provided) page 7,8 (sections C.l and 3.)

Proposed references to be provided to applicants during examination: NONE Learning Objective: (As available) i-l Question Source: Bank #

Modified Bank # (Note changes or attach parent)

New X Question History: Last NRC Exam Question Cognitive Level: Memory or Fundamental Knowledge Comprehension or Analysis X 10 CFR Part 55 Content: 55.41 (7) 55.43 Comments:

Page 24 of 203

ES-401 Sample Written Examination Form ES-401-5 Question Worksheet

- l /Examination Outline Cross-reference: Level RO SRO Tier # 2 Group # 1 WA # 217000 A2.09 importance Rating 2.9 Ability to (a) predict the impacts of the following on the Reactor Core Isolation Cooling System (RCIC);and (b) based on those predications, use procedures to correct, control, or mitigate the consequences of those abnormal conditions or operations: Loss of Vacuum Pump.

Proposed Question: Common 13 RClC was placed in service for its periodic surveillance test in accordance with PNPS 8.5.5.1 RCIC Pump Quarterly Flow Test.

Condensate and non-condensablesfrom the RClC barometric condenser are drawn through a strainer into the vacuum tank due to the differential pressure created by the vacuum pump.

For the surveillance test the vacuum pump -(I) and if the vacuum pump failed during the test, RClC -(2) trip due to high vacuum tank pressure.

>L/ A. (1) must be manually started (2) would B. (1) must be manually started (2) would not C. (1) auto starts (2) would D. (I) auto starts (2) would not Page 25 of 203

ES-401 Sample Written Examination Form ES-401-5 Question Worksheet Proposed Answer: B i_/ Explanation (Optional):

A. Incorrect. High vacuum tank pressure is not a trip signal.

6. Correct. The vacuum pump starts automatically on a RClC initiation; it can also be controlled from a three-position switch (START, AUTO, STOP) on panel 904 that also has a PULL-TO-LOCK feature. In addition, the pump can be controlled (ON, OFF) by a pushbutton located in the RClC room. It is powered from 125 VDC panel D-7. Once the pump is started, it must be manually stopped. PNPS 8.5.5.1, RCIC Pump Quarterly Flow Test, Att.1, step [6],has the operator start the pump manually for the test. A control room alarm will sound on panel 904 if vacuum tank pressure is high at 5 Hg and decreasing.

C. Incorrect. Auto starts on a RClC initiation signal only.

D. Incorrect. Auto starts on a RClC initiation signal only.

Technical Reference(s) RClC reference text, Rev.4, (Attach if not previously provided) page 17 section C.7 and page 22 section D.4.

PNPS 8.5.5.1, Rev.56, Att.1, step[6]

Proposed references to be provided to applicants during examination: NONE 4 Learning Objective: (As available)

Question Source: Bank #

Modified Bank # (Note changes or attach parent)

New X Question History: Last NRC Exam Question Cognitive Level: Memory or Fundamental Knowledge Comprehension or Analysis X 10 CFR Part 55 Content: 55.41 (5) 55.43 Comments:

Page 26 of 203

ES-401 Sample Written Examination Form ES-401-5 Question Worksheet l/

Examination Outline Cross-reference: Level RO SRO Tier # 2 Group # 1 WA # 218000 2.1.27 Importance Rating 2.8 ADS - Conduct of Operations: Knowledge of system purpose andlor function.

Proposed Question: Common 14 Due to a plant transient an Automatic Depressurization System (ADS) blowdown is in p rogress.

ADS serves as a backup to the (I) under -(2) break loss-of-coolant accident conditions.

A. (I) High Pressure Coolant Injection System (HPCI).

(2) Large.

B. (I) Low Pressure Coolant Injection (LPCI) or Core Spray (CS) systems.

(2) Small.

c C. (1) High Pressure Coolant Injection System (HPCI).

(2) Small.

D. (1) Low Pressure Coolant Injection (LPCI) or Core Spray (CS) systems.

(2) Large.

Proposed Answer: C Explanation (Optional):

A. Incorrect. (2) wrong.

B. Incorrect. (1) wrong.

C. Correct. (1) The automatic depressurizationsystem (ADS), in conjunction with the low pressure core standby cooling systems, serves as a backup to the (2) high pressure coolant injection (HPCI) system if it cannot maintain vessel water level during a small break loss-of-coolant accident D. Incorrect. (2) wrong.

Technical Reference(s) ADS Reference text, Rev.3, (Attach if not previously provided) section B.1. (page 6), section 4

Page 27 of 203

ES-401 Sample Written Examination Form ES-401-5 Question Worksheet C.3. (page 14)

Proposed references to be provided to applicants during examination: NONE Learning Objective: (As available)

Question Source: Bank #

Modified Bank # (Note changes or attach parent)

New X Question History: Last NRC Exam Question Cognitive Level: Memory or Fundamental Knowledge Comprehension or Analysis X 10 CFR Part 55 Content: 55.41 (7) 55.43 Comments:

Page 28 of 203

ES-401 Sample Written Examination Form ES-401-5 Question Worksheet

-d Examination Outline Cross-reference: Level RO SRO Tier # 2 Group # 1 KIA # 223002 K6.06 Importance Rating 2.8 Knowledge of the effect that a loss or malfunction of the following will have on PCIS: Various Process Instrumentation.

Proposed Question: Common 15 One of the Reactor Low Pressure and One of the High Drywell Pressure sensing instruments that provide inputs to PClS have failed and have been placed in their tripped position.

(1) Which PClS groups are provided inputs by these parameters?

(2) How would the PClS groups respond to these instruments being in the tripped co ndition?

A. (1) Groups 2 and 7 have Reactor Low Pressure inputs. Groups 1,2, and 3 have inputs from High Drywell Pressure.

(2) No PClS group isolations would have occurred because the logic is always one-out-of-two-taken-twice.

B. (I) Groups 4 and 7 have Reactor Low Pressure inputs. Groups 1, 2,3 and 7 have inputs from High Drywell Pressure.

(2) Group 7 would have isolated. No other PClS group isolations would have occurred because their logic is always one-out-of-two-taken-twice.

C. (1) Groups 3 and 4 have Reactor Low Pressure inputs. Groups 2,3 and 7 have inputs from High Drywell Pressure.

(2) Group 7 would have isolated. No other PClS group isolations would have occurred because their logic is always one-out-of-two-taken-twice.

D. (1) Groups 4 and 7 have Reactor Low Pressure inputs. Groups 2,3 and 7 have inputs from High Drywell Pressure.

(2) No PClS group isolations would have occurred because the logic is always 0ne-out-of-two-taken-twice.

Page 29 of 203

ES-401 Sample Written Examination Form ES-401-5 Question Worksheet Proposed Answer: D 2 Explanation (Optional):

A. Incorrect. Wrong groups for given signals.

B. Incorrect. Wrong groups for HI DW press. No isolations would have occurred.

C. Incorrect. Wrong groups for Low Reactor pressure, No isolations would have occurred.

D. Correct. Per FSAR table and reference text.

Technical Reference(s) FSAR Table 5.2.4, PClS (Attach if not previously provided)

Reference Text, Rev.3, section B.l .g.

Proposed references to be provided to applicants during examination:

Learning Objective: (As available)

Question Source: Bank #

Modified Bank # (Note changes or attach parent)

New X Question History: Last NRC Exam

\.dQuestion Cognitive Level: Memory or Fundamental Knowledge Comprehension or Analysis X 10 CFR Part 55 Content: 55.41 (5) 55.43 Comments:

Page 30 of 203

ES-401 Sample Written Examination Form ES-401-5 Question Worksheet Examination Outline Cross-reference: Level RO SRO Tier # 2 Group # 1 KIA # 239002 2.2.25 Importance Rating 2.5 Equipment Control: Knowledge of the bases in technical specifications for LCOs and safety limits (SRVs).

Proposed Question: Common 16 Regarding the Over Pressure relief capability of the primary system; the combination of safety valves and safety relief valves are designed to limit transient reactor pressure to less than or equal to -(I)- . Additional margin is provided by assuming-(2) .

A. (1) 1400 psig (2) one valve fails to function

8. (1) 110% of design pressure (2) an Indirect Reactor Scram C. (1) 1240 psig (2) an Indirect Reactor Scram

'L' D. (1) 125% of design pressure (2) one valve fails to function Proposed Answer: B Explanation (Optional):

A. Incorrect. Per TS bases.

6. Correct. Per TS bases 3.6.D.

C. Incorrect. Per TS bases.

D. Incorrect. Per TS bases.

Technical Reference(s) TS Bases 3.5.E (Attach if not previously provided) v Page 31 of 203

ES-401 Sample Written Examination Form ES-401-5 Question Worksheet Proposed references to be provided to applicants during examination: NONE Learning Objective: (As available)

Question Source: Bank #

Modified Bank # (Note changes or attach parent)

New X Question History: Last NRC Exam Question Cognitive Level: Memory or Fundamental Knowledge X Comprehension or Analysis 10 CFR Part 55 Content: 55.41 55.43 (2)

Comments:

Page 32 of 203

ES-40I Sample Written Examination Form ES-401-5 Question Worksheet L./

Examination Outline Cross-reference: Level RO SRO Tier # 2 Group # 1 KIA # 239002 K5.01 - SRVS Importance Rating 3.4 Knowledge of the operational implications of the following as they apply to ReliefKafety Valves: Relief function of SRV operation.

Proposed Question: Common 17 The reactor is operating at 100% power when a small transient occurs causing reactor power and pressure to lower. A small reactor level swell is also observed. Reactor power and pressure then return to approximately their original values and conditions stabilize.

Which ONE (I) of the following would cause this transient to occur?

A. A single reactor recirculation pump runback B. A control rod drifting in.

C. Loss of power to the EPR causing an auto swap to the MPR.

J-D. A safety relief valve going open and then reclosing.

Proposed Answer: D Explanation (Optional):

A. Incorrect. Once the pump runback occurred, reactor power would remain lower.

B. Incorrect. Once the rod drifted in, reactor power would remain lower.

C. Incorrect. Reactor pressure will increase slightly, not lower.

D. Correct. SRV lift will cause reactor pressure and power to lower with an associated reactor level swell. Once. closed conditions will stabilize.

Technical Reference(s) PNPS 2.4.29 - Stuck open (Attach if not previously provided)

SRV.

Proposed references to be provided to applicants during examination: NONE Learning Objective: (As available)

Page 33 of 203

ES-401 Sample Written Examination Form ES-401-5 Question Worksheet L/ Question Source: Bank # #40 abnormal emergency Modified Bank # (Note changes or attach parent)

New Question History: Last NRC Exam Question Cognitive Level: Memory or Fundamental Knowledge Comprehension or Analysis X 10 CFR Part 55 Content: 55.41 (5) 55.43 Comments:

Page 34 of 203

ES-401 Sample Written Examination Form ES-401-5 Question Worksheet 4

Examination Outline Cross-reference: Level RO SRO Tier # 2 Group # 1 KIA # 295002 K4.12 Importance Rating 3.5 Knowledge of the reactor water control system design features and or interlocks which provide for the following: manual and auto control of the system.

Proposed Question: Common 18 0 The plant is at 100% power.

0 All systems are operable.

FWLC is in its normal full power configuration.

As you are monitoring control room indications the Total Feed Flow signal to the Feedwater Level Control System (FWLC) fails low.

The FWLC system will automatically cause reactor vessel level to -(I)- due to a steam flow/feed flow mismatch and reactor vessel level wi11-(2)- .

..4 A. ( I ) Rise (2) Continue to rise to the Main Turbine Trip setpoint unless the operators take manual control of the system.

B. (1) Rise (2) Continue to rise but then stabilize at a higher value below the Main Turbine Trip setpoint.

C. (I) Lower (2) Continue to lower to the Low Level Reactor Scram unless the operators take manual control of the system.

D. (1)Lower (2) Continue to lower but stabilize above the Low Level Reactor Scram setpoint.

Page 35 of 203

ES-401 Sample Written Examination Form ES-401-5 Question Worksheet Proposed Answer: A d,

Explanation (Optional):

A. Correct. A loss of the total FW flow signal causes a large feed flowkteam flow error (640-5) to develop. This error results in the FRVs opening in an attempt to restore the erroneous signal to its initial value. The following actions will occur:

a)A recirc pump runback will occur due to Nv flow erroneously indicating ~ 2 0 % .

b) The RWM RPSP is activated.

Actual RPV level will increase and the following will occur.

a) A high RPV water level alarm at +32 inches.

b) A turbine trip at +45 inches B. Incorrect. Level will not stabilize below the trip setpoint.

C. Incorrect. Level increases.

D. Incorrect. Level increases.

Technical Reference(s) FWLC Reference Text, Rev.5, (Attach if not previously provided) page 39, section F.3.e.

Proposed references to be provided to applicants during examination: NONE Learning Objective: (As available)

LJ Question Source: Bank #

Modified Bank # (Note changes or attach parent)

New X Question History: Last NRC Exam Question Cognitive Level: Memory or Fundamental Knowledge Comprehension or Analysis X 10 CFR Part 55 Content: 55.41 7 55.43 Comments:

Page 36 of 203

ES-401 Sample Written Examination Form ES-401-5 Question Worksheet Examination Outline Cross-reference: Level RO SRO 4

Tier # 2 Group # 1 WA # 261000A2.12 Importance Rating 3.2 Ability to predict the impacts on the following (SBGT) and based on those predictions use procedures to correct, control, or mitigate the consequences of those abnormal conditions or operations: Hi FP vent rad.

Proposed Question: Common 19 Refueling is in progress. You are in contact with the refuel floor SRO from the main control room. As a fuel bundle is being moved from the fuel pool to the vessel control room alarms annunciate indicating that Refueling Floor Ventilation Exhaust Rad Monitors for channels A and B are in High Alarm.

Control room indications now show that ONLY the SBGT Fan A is in service. The current switch positions for each fan are:

I SBGT Fan A -AUTO 0 SBGT Fan B - MAINT SBGT system flow indicates 1900 CFM.

,4 In accordance with PNPS 2.2.50 Standby Gas Treatment (SBGT), (1) what actions are you required to take, if any, based on the above information? AND (2) What is the reason for your decision?

A. (1) Start SBGT Fan B (2) Because it failed to immediately auto start due to its control switch position being in MAINT.

B. (1) No action required.

(2) SBGT Fan B was not required and did not auto start because system flow was adequate.

C. (1) Start SBGT Fan B (2) Because it initially failed to auto start on low system flow ( ~ 2 0 0 0cfm).

D. (I) Start SBGT Fan B (2) Because it should have auto started on the initiation signal.

Page 37 of 203

ES-401 Sample Written Examination Form ES-401-5 Question Worksheet

-J Proposed Answer: D Explanation (Optional):

A. Incorrect. With control switch in MAINT fan will still auto start on an initiation signal.

B. Incorrect. Fan should have auto started regardless of system flow.

C. Incorrect. Fan failed to immediately auto start on the initiation signal, no low flow.

D. Correct. If the control switch is in STBY when an initiation signal is received, B train inlet damper opens. When OPEN limit switch picks up, B fan starts. Fan start circuit energizes heaters and opens B outlet damper. After 65 seconds, the B train shuts down by closing the inlet damper, which secures the fan, which de-energizes the heaters and closes the outlet damper. If SBGT flowrate is sensed to be ~ 2 0 0 0 scfm, the B train will re-start. When the initiation signal is reset, the inlet damper closes, which secures the fan, which de-energizes the heaters and closes the outlet damper.

If the control switch is in MAINT it is the same as STBY except that the B train will not shutdown after 65 seconds. It will continue to run until initiation signal is reset or the fan control switch is taken to OFF.

Technical Reference(s) SBGT Reference text, Rev.9, (Attach if not previously provided) page 17, section D.5.

PNPS 2.2.50, Rev.61 step 7.2[2].

PNPS 1.3.34. - If an auto action doesnt happen, make it happen.

Proposed references to be provided to applicants during examination: NONE Learning Objective: (As available)

Question Source: Bank #

Modified Bank # (Note changes or attach parent)

New X Question History: Last NRC Exam Question Cognitive Level: Memory or Fundamental Knowledge Comprehension or Analysis X 10 CFR Part 55 Content: 55.41 7 55.43 Page 38 of 203

ES-401 Sample Written Examination Form ES-401-5 Question Worksheet Comments:

--4' Page 39 of 203

ES-401 Sample Written Examination Form ES-401-5 Question Worksheet

.4 Examination Outline Cross-reference: Level RO SRO Tier # 2 Group # 1 WA # 262001 K1.03 Importance Rating 3.4 Knowledge of the physical connections and/or cause effect relationships between AC Dist. and Offsite power.

Proposed Question: Common 20 A Loss of Offsite power has occurred.

In accordance with station procedures, if the Station Blackout Diesel is the only power source available, a safe plant shutdown will be assured because the Station Blackout Diesel A. Will automatically start and can be manually aligned to provide power simultaneously to both the A-5 AND A-6 4160V emergency service busses.

B. Will automatically start and provide power to either the A-5 OR A-6 41 60V emergency service busses.

.L.---,

C. Can be manually started and then can be manually aligned to provide power to the A-5 OR A-6 4160V emergency service bus.

D. Can be manually started and then can be manually aligned to provide power simultaneously to BOTH the A-5 and A-6 41 60V emergency service busses.

Proposed Answer: C Explanation (Optional):

A. Incorrect. There are no auto start features associated with the SBO diesel.

B. Incorrect. There are no auto start features associated with the SBO diesel.

C. Correct. The station blackout diesel generator fulfills the NRC requirement for an Alternate AC Power source to be used in the case of a station blackout. The SBO diesel can be manually started (no auto start feature) and aligned to either A-5 or A-6 within 10 minutes to supply limited AC power.

D. Incorrect. The SBO diesel can be manually aligned to either A-5 OR A-6.

Page 40 of 203

ES-401 Sample Written Examination Form ES-401-5 Question Worksheet Technical Reference@) 4160 V Distribution System, (Attach if not previously provided)

W Reference Text, Rev. 1, Page 8, section 6.4.

Proposed references to be provided to applicants during examination: NONE Learning Objective: (As available)

Question Source: Bank #

Modified Bank # PNPS 3240 (Note changes or attach parent)

New Question History: Last NRC Exam Question Cognitive Level: Memory or Fundamental Knowledge Comprehension or Analysis X 10 CFR Part 55 Content: 55.41 7 55.43 Comments:

\/ 1211 - Gil - add to distractor A can be manually aligned 12/19- Steve - revised Page 41 of 203

ES-401 Sample Written Examination Form ES-401-5 Question Worksheet i_/ Examination Outline Cross-reference: Level RO SRO Tier # 2 Group # 1 WA # 262002 A2.01 Importance Rating 2.6 Ability to predict the impacts of the following on UPS and based those predictions use proceduresto correct, control or mitigate the consequences of those abnormal conditions: undervoltage.

Proposed Question: Common 21 Reactor power is 90% with the 250 VDC supply breaker (D1022)to the Vital MG set open for maintenance. While in this configuration the following sequence of events occur:

0 Multiple control room alarms annunciate including o RPlS INOP (C905L-D4) o MG A SCOOP TUBE LOCK UP (C904RC-A6) o MG B SCOOP TUBE LOCK UP (C904RC-A5) 0 Operators report that the 120 VAC Distribution Status light for Y2 has extinguished and that it illuminates after a brief time delay.

0 Alarm Y2 AUTOMATIC TRANSFER (C3RC-A2) annunciates c

, The 905 operator then reports that RPV level is +32 inches and slowly rising.

The procedurally authorized actions for stabilizing RPV level is:

A. Lower steam demand by resetting the RECIRC MG set scoop tubes and reducing recirc pump speed.

6. Lower steam demand by establishing local control of the RECIRC MG sets and reducing recirc pump speed.

C. Place BOTH Feedwater Regulating Valve Transfer Stations to MANUAL and reduce feed flow D. Set the Feedwater Master Level Controller set point to current RPV level and then depress the feedwater regulating LOCKUP RESET pushbuttons Proposed Answer: D Explanation (Optional):

Comment: PNPS does not utilize conventional UPS devices for control of any operationally significant equipment. PNPS however does utilize a Vital Services bus that is powered from an

-A AC source, a DC source and has an automatic transfer feature to a reserve power supply which Page 42 of 203

ES-401 Sample Written Examination Form ES-401-5 Question Worksheet is equivalent to a conventional UPS.

J A. Incorrect because although the Recirc M G sets are locked up, the momentary loss of Y-2 has resulted in lockups on both feed reg valves. Lowering recirc pump speed will aggravate the rising level trend and is not procedurally directed.

B. Incorrect because although the Recirc MG sets are locked up, the momentary loss of Y-2 has resulted in lockups on both feed reg valves. Lowering recirc pump speed will aggravate the rising level trend and is not procedurally directed.

C. Incorrect because taking manual control of the M/A stations will not affect feedflow with the valves locked up D. Correct - is the procedurally correct response.

Proposed references to be provided to applicants during examination: NONE Learning Objective: (As available)

.d Question Source: Bank #

Modified Bank # (Note changes or attach parent)

New X Question History: Last NRC Exam Question Cognitive Level: Memory or Fundamental Knowledge Comprehension or Analysis X 10 CFR Part 55 Content: 55.41 4 55.43 Comments:

Page 43 of 203

ES-401 Sample Written Examination Form ES-401-5 Question Worksheet d Examination Outline Cross-reference: Level RO SRO Tier # 2 2 Group # 1 1 WA # 263000 K2.01 - DC Elec.

Dist.

Importance Rating 3.1 Knowledge of Electrical Power supplies to the following: Major D.C. loads.

Proposed Question: Common 22 The plant is at 100% power. All systems are operating normally when a loss of all 250 VDC power occurs.

Which of the following Containment Isolation Valves is affected by the loss of 250 VDC power?

A. RClC Pump Suction from the CST, MO-1301-22.

B. HPCl Inboard Steam Supply Isolation valve, MO-2301-4.

-d C. RHR Pump A Torus Suction valve, MO-1001-7A D. HPCl Outboard Steam Supply Isolation Valve MO-2301-5 Proposed Answer: D Explanation (Optional):

A. Incorrect. Not powered from 250 VDC.

B. Incorrect. Not a containment isolation valve powered from 250 VDC.

C. Incorrect. Not a containment isolation valve powered from 250 VDC.

D. Correct. 250 VDC loss affects the containment isolation system because there are two 250 VDC containment isolation valves. The valves are the RHR shutdown cooling suction valve, MO-1001-47 (normally closed with breaker open), and the HPCl steam supply isolation valve MO-2301-5, (normally open).

Technical Reference(s) 250VDC System Reference, (Attach if not previously provided)

Rev.5, page I O , section E.3.

Proposed references to be provided to applicants during examination: NONE Page 44 of 203

ES-401 Sample Written Examination Form ES-401-5 Question Worksheet Learning Objective: (As available) 4 Question Source: Bank #

Modified Bank # (Note changes or attach parent)

New X Question History: Last NRC Exam Question Cognitive Level: Memory or Fundamental Knowledge Comprehension or Analysis X 10 CFR Part 55 Content: 55.41 7 55.43 Comments:

12/1 - change distractors from all HPCl valves 12/19 - Steve - revised Page 45 of 203

ES-401 Sample Written Examination Form ES-401-5 Question Worksheet L./ Examination Outline Cross-reference: Level RO SRO Tier # 2 Group # I KIA # 264000 K3.03 Importance Rating 4. I Knowledge of the effect that a loss or malfunction of the EDGs will have on the following: Major loads powered by buses fed from the EDGs.

Proposed Question: Common 23 When making rounds through the plant an operator observes that all four isolation switches for the B EDG have been taken to local position.

Under these conditions the B EDG will -(I)-. .

A. Automatically start in the droop mode on a LOCA signal ONLY.

8. Automatically start and load in the droop mode on a LOCA signal ONLY.

C. Have to be manually started and loaded.

J D. Automatically start and load in the isochronous mode on a LOOP signal ONLY.

Proposed Answer: D Explanation (Optional):

A. Incorrect.- only starts on a LOOP.

I1 B. Incorrect. - I 11 I C. Incorrect.-will still auto start on a LOOP.

D. Correct. - per PNPS 2.4.1.43 App. E, page 1 Technical Reference(s) EDG Reference Text, Rev. 9, (Attach if not previously provided) page 34 PNPS 2.4.1.143, Rev.36, App.

E. , sht 1.

Proposed references to be provided to applicants during examination:

,.-/

Page 46 of 203

ES-401 Sample Written Examination Form ES-401-5 Question Worksheet Learning Objective: (As available)

'W Question Source: Bank # EDGs #52 Modified Bank # (Note changes or attach parent)

New Question History: Last NRC Exam Question Cognitive Level: Memory or Fundamental Knowledge Comprehension or Analysis X 10 CFR Part 55 Content: 55.41 7 55.43 Comments:

Page 47 of 203

ES-40I Sample Written Examination Form ES-401-5 Question Worksheet d Examination Outline Cross-reference: Level RO SRO Tier # 2 Group # I WA # 300000 K5.01 Importance Rating 2.5 Knowledge of the operational implications as they apply to the following: lnst Air Compressors.

Proposed Question: Common 24 The plant is operating at 100% with the K-I 17 air compressor out of service.

While in this configuration, instrument air pressure begins to degrade. If pressure continues to degrade the sequence of events affecting plant operation will be?

A. (1) AO-4365 shuts to isolate non-essential instrument air header (2) AO-4350 shuts to isolate the service air header (3) Feedwater control valves lock up (4) MSlVs begin to drift closed B. (1) AO-4350 shuts to isolate the service air header L//

(2) AO-4365 shuts to isolate non-essential instrument air header (3) MSlVs begin to drift closed (4) Feedwater control valves lock up C. ( I )AO-4350 shuts to isolate the service air header (2) AO-4365 shuts to isolate non-essential instrument air header (3) Scram Pilot valve air header dump valve opens (4) Feedwater control valves lock up D. (1) AO-4350 shuts to isolate the service air header (2) AO-4365 shuts to isolate non-essential instrument air header (3) Feedwater control valves lock up (4) Scram Pilot valve air header dump valve opens Proposed Answer: D Explanation (Optional):

A. Incorrect. Incorrect sequence.

B. Incorrect. Incorrect sequence.

bl C. Incorrect. Incorrect sequence.

Page 48 of 203

~ ES-401 Sample Written Examination Form ES-401-5 Question Worksheet D. Correct. When instrument air pressure degrades, the following automatic actions occur:

1. 102 psig Lead compressor K l 1l / K l l O loads L/ Lagging K-I 1O/K-I 11 compressor starts and loads
2. 93 psig:
3. 90 psig: K-I 17 starts and loads. Unloads at 110 psig.
4. 85 psig: AO-4350 shuts to isolate the service air header AND AO-4353 shuts to isolate instrument air from low pressure service air (normally shut)
5. 80 psig: AO-4365 shuts to isolate non-essential instrument air header
6. 75 psig: Feedwater control valves lock up
7. 55 psig: Scram Pilot valve air header dump valve opens causing a Reactor Scram Technical Reference(s) High Press. Air System (Attach if not previously provided) reference text, Rev. 9, page 24, section E.

Proposed references to be provided to applicants during examination:

Learning Objective: (As available)

Question Source: Bank #

Modified Bank # (Note changes or attach parent)

New X b-Question History: Last NRC Exam Question Cognitive Level: Memory or Fundamental Knowledge X Comprehension or Analysis I O CFR Part 55 Content: 55.41 4 55.43 Comments:

Page 49 of 203

ES-401 Sample Written Examination Form ES-401-5 Question Worksheet

\-* Examination Outline Cross-reference: Level RO SRO Tier # 2 Group # I KIA # 4000000 A2.03 Importance Rating 2.9 Ability to predict the impacts of the following on CCWS and based on those predictions use procedures to correct control or mitigate the consequences of those abnormal operation: Highhow CCW temps.

Proposed Question: Common 25 The reactor is at 100% power with the B TBCCW pump out of service. The A TBCCW pump then trips and cant be restarted.

In addition to scramming the reactor, which ONE (I) of the following groups of actions are the operators required to take in accordance with PNPS 2.4.41?

A. Trip the Main Turbine.

Trip all Feedwater Pumps and all but one Condensate Pump.

Place HPCI in level control as needed.

Place RCIC in pressure Control.

B. Trip the Main Turbine.

Trip all Feedwater Pumps and all but one Condensate Pump.

Place RCIC in level control as needed.

Place HPCI in pressure Control.

C. Trip all Feedwater Pumps and all but one Condensate Pump.

Place HPCl in level control as needed.

Place RClC in pressure Control.

D. Trip all Feedwater Pumps and all but one Condensate Pump.

Place RClC in level control as needed.

Place HPCI in pressure Control.

Proposed Answer: D Explanation (Optional):

A. Incorrect, Per PNPS 2.4.41 - Main turbine is not tripped but is allowed to trip on its own.

LJ Page 50 of 203

ES-401 Sample Written Examination Form ES-401-5 Question Worksheet

8. Incorrect. Per PNPS 2.4.41 - Main turbine is not tripped but is allowed to trip on its own.

-d C. Incorrect. HPCI is for pressure control, RCIC for level control.

D. Correct. Per PNPS 2.4.31, Rev. 24, step 4.0[1]

Technical Reference(s) PNPS 2.4.41, Rev. 24, step (Attach if not previously provided) 4.0[1]

Proposed references to be provided to applicants during examination: NONE Learning Objective: (As available)

Question Source: Bank #

Modified Bank # (Note changes or attach parent)

New X Question History: Last NRC Exam Question Cognitive Level: Memory or Fundamental Knowledge X Comprehension or Analysis 10 CFR Part 55 Content: 55.41 10 55.43 Comments:

Page 51 of 203

ES-401 Sample Written Examination Form ES-401-5 Question Worksheet

~d Examination Outline Cross-reference: Level RO SRO Tier # 2 Group # I KIA # 400000 K1.02 Importance Rating 3.2 Component Cooling Water - Knowledge of the Physical connections and/or cause effect relationships between CCWS and the following: Loads cooled by CCWS.

Proposed Question: Common 26 A loss of loop B Reactor Building Closed Cooling Water (RBCCW) pumps occurs. Ten minutes later the plant conditions are as follows:

0 Reactor is scrammed.

0 Drywell temperature is > 250 degrees F. and rising.

Drywell pressure is 8 psig and stable.

Before cross tying RBCCW to supply essential loads, the loop B non-essential isolation valves are shut in order to  ?

A. Prevent runout of the RBCCW Loop A pumps.

u-I B. Ensure sufficient cooling to essential RBCCW Loop B loads.

C. Prevent piping failure from water hammer.

D. Prevent excessive cooldown of containment.

Proposed Answer: C Explanation (Optional):

A. Incorrect. Not a concern, flow thru the heat exchangers is controlled via procedure 2.2.1 9.

Att. I- Maximize RBCCW cooling.

B. Incorrect. Not a concern, cooling is controlled via procedure 2.2.19. Att.1- Maximize RBCCW cooling.

C. Correct. At elevated DW temperatures (>250) boiling could occur in the DW coolers causing condensation induced water hammer if the non essential valves are not closed first.

D. Incorrect. Cross tie is done ensure containment cooling is maximized.

Page 52 of 203

ES-401 Sample Written Examination Form ES-401-5 Question Worksheet Technical Reference(s) PNPS 2.4.42, Rev.27Loss of (Attach if not previously provided)

W RBCCW, section 4.2 caution statement PNPS 2.2.19.5, RHR modes of operation for transients Rev.19, Att. I.

Proposed references to be provided to applicants during examination: NONE Learning Objective: (As available)

Question Source: Bank # RBCCW-10563 Modified Bank # (Note changes or attach parent)

New Question History: Last NRC Exam Question Cognitive Level: Memory or Fundamental Knowledge Comprehension or Analysis X 10 CFR Part 55 Content: 55.41 4, 10

.u 55.43 Comments:

Page 53 of 203

ES-401 Sample Written Examination Form ES-401-5 Question Worksheet d

Examination Outline Cross-reference: Level RO SRO Tier # 2 Group # 2 KIA # 201 002 2.1.28 Importance Rating 3.2 Knowledge of the system purpose or function of major components and controls.

Proposed Question: Common 27 You are inserting control rods for a rapid power reduction and the shift manager authorizes use of the Emergency In control switch.

Which ONE (I) of the following describe an advantage of using the switch for a rapid power reduction.

A. Bypasses the insert control rod block function of the RWM.

B. Bypasses the control rod drift circuitry.

C. Bypasses the settle function.

J, .~>

D. Bypasses the stabilizing valve function.

Proposed Answer: C Explanation (Optional):

A. Incorrect. The switch does not bypass that function.

B. Incorrect. Rod drift circuitry is not tied into the use of the switch. Rod drift occurs if any rod is not in an even rod position and that rod is not selected and driving.

C. Correct. Per RMCS reference text - The EMERG IN position bypasses all interlocks, allowing selected rod insertion unless a rod worth minimizer (RWM) insert block is present. The switch acts directly on the selected CRD directional control valves and bypasses the rod sequence timer. When this position is used, the CRD has no settle function and forces water past seals in the CRD while settling into a notch.

D. Incorrect. Stabilizing valves still must function to permit rod movement.

Page 54 of 203

ES-401 Sample Written Examination Form ES-401-5 Question Worksheet Technical Reference(s) RMCS Reference text, Rev.1, (Attach if not previously provided)

'.~../ step C.5.

PNPS 2.2.88, Rev.25, section 7.5 Proposed references to be provided to applicants during examination:

Learning Objective: (As available)

Question Source: Bank # RPlS

  1. 6114 Modified Bank # (Note changes or attach parent)

New Question History: Last NRC Exam Question Cognitive Level: Memory or Fundamental Knowledge X Comprehension or Analysis 10 CFR Part 55 Content: 55.41 6 55.43

....+A' Comments:

Page 55 of 203

ES-401 Sample Written Examination Form ES-401-5 Question Worksheet d Examination Outline Cross-reference: Level RO SRO Tier # 2 Group # 2 WA # 239001 K5.05 Importance Rating 2.8 Knowledge of the operational implications of the following concepts as they apply to Main Steam system: Flow Indication.

Proposed Question: Common 28 The plant is operating at 90% power. On routine control room rounds you notice Main Steam Line Flow meter FI 640-23 A is reading significantly higher than the other three indicators. All other control room indications are normal. All equipment is operable.

The following alarms are then received.

a MAIN STEAM LINE FLOW HI - Panel C905L B-I MAIN STEAM LINE FLOW HI - Panel C905L B-2 The plant response to this condition is that u, A Group 1 isolation should have occurred and then a Reactor Scram should have A.

occurred.

B. The Feedwater Level Control System would sense an increased main steam flow causing it to increase feedwater flow to the reactor vessel. No other actions would occur.

C. No automatic actions should have occurred. Only one half of the logic to PClS is tripped for actuating a Group 1 isolation.

D. ONLY a Group 1 isolation should have occurred.

Proposed Answer: A Explanation (Optional):

A. Correct. Alarms in, logic has been made up in PClS for a Group 1 isolation (MSIV closure will result in reactor scram).

B. Incorrect. Although steam flow is an input to FLCS, the GROUP1 isolation and scram will prevent a feed flow increase.

I/ C. Incorrect. Group 1 isolation and scram will occur.

Page 56 of 203

ES-401 Sample Written Examination Form ES-401-5 Question Worksheet D. Incorrect. Group 1 isolation and scram will occur.

Technical Reference(s) Main Steam Reference Text, (Attach if not previously provided)

Rev.3 , page 15, section C.4.

Proposed references to be provided to applicants during examination: NONE Learning Objective: (As available)

Question Source: Bank #

Modified Bank # (Note changes or attach parent)

New X Question History: Last NRC Exam Question Cognitive Level: Memory or Fundamental Knowledge X Comprehension or Analysis 10 CFR Part 55 Content: 55.41 7 55.43 Comments:

Page 57 of 203

ES-401 Sample Written Examination Form ES-401-5 Question Worksheet

J Examination Outline Cross-reference
Level RO SRO Tier # 2 Group # 2 KIA # 201006 A I .01 Importance Rating 3.2 Ability to predict and/or monitor changes in parameters associated with operating the ROD WORTH MINIMIZER SYSTEM controls including: Rod Position.

Proposed Question: Common 29 A central control rod is being moved from position 32 to 34 during a startup just prior to placing the turbine online.

When the control rod reaches position 34, the reed switch is faulty and does not close.

Which ONE (1) of the following describes how the Rod Worth Minimizer (RWM) will respond to this faulty reed switch?

A. The RWM will enforce both a control rod insert and withdraw block.

\ - )

6. The RWM will enforce a control rod withdraw block ONLY.

C. The RWM will enforce a control rod insert block ONLY.

D. The RWM will display a select error ONLY.

Proposed Answer: A Explanation (Optional):

A. Correct. When the RWM detects a rod at a position with a faulty reed switch (equipment failure), it causes both an insert and withdraw block.

B. Incorrect. Also enforces insert block.

C. Incorrect. Also enforces insert block.

D. Incorrect. A select error occurs whenever the operator selects a rod other than one contained in the currently latched step. A select error will also occur when a non-error rod is selected with a withdraw error or three insert errors present.

Technical Reference(s) RWM Reference Text, Rev.3, (Attach if not previously provided) page 17. Section F.l .a lJ Page 58 of 203

ES-401 Sample Written Examination Form ES-401-5 Question Worksheet

-,J Proposed references to be provided to applicants during examination: NONE Learning Objective: (As available)

Question Source: Bank # PNPS -

6109 Modified Bank # (Note changes or attach parent)

New Question History: Last NRC Exam Question Cognitive Level: Memory or Fundamental Knowledge Comprehension or Analysis X 10 CFR Part 55 Content: 55.41 6 55.43 Comments:

1211 - Gil - change to different power level 12/19- Steve - changed I -d Page 59 of 203

ES-401 Sample Written Examination Form ES-401-5 Question Worksheet

.v Examination Outline Cross-reference: Level RO SRO Tier # 2 Group # 2 KfA ## 204000 K6.09 -

Importance Rating 2.7 Knowledge of the effect that a loss or malfunction of the following will have on the RWCU system: CRD hydraulics.

Proposed Question: Common 30 The plant is cold shutdown with RWCU in the reject mode of operation and CRD providing makeup in accordance with PNPS 2.2.83, Reactor Cleanup System, for vessel level control.

Reactor water vessel level is currently at +30 inches.

A failure then occurs due to a loss of air to the in service CRD flow control valve.

If no operator action is taken, Reactor Water Vessel Level will L..-d, A. Be maintained at 30 because RWCU automatically increased reject flow to account for the increased flow from the CRD flow control valve failing open.

B. Be maintained at 30 because RWCU automatically decreased reject flow to account for the decreased flow from the CRD flow control valve failing closed.

C. Will increase because the CRD flow control valve failed open. RWCU reject flow has no automatic response to increasing level.

D. Will decrease because the CRD flow control valve failed closed. RWCU reject flow has no automatic response to decreasing level.

Page 60 of 203

ES-401 Sample Written Examination Form ES-401-5 Question Worksheet Proposed Answer: D

,u Explanation (Optional):

A. Incorrect. The reject flow control valve CV-1239 has no automatic functions.

B. Incorrect. The reject flow control valve CV-1239 has no automatic functions.

C. Incorrect. The CRD flow control valve fails closed.

D. Correct. RWCU reject has no auto functions. CRD flow control valve fails closed. Lees makeup with same reject will result in a decreasing vessel level.

Technical Reference(s) PNPS 2.4.1 1.I ,Rev.l9, section (Attach if not previously provided) 5.4.

PNPS 2.2.83, Rev.94, section 7.3. Ir61 Proposed references to be provided to applicants during examination: NONE Learning Objective: (As available)

Question Source: Bank #

Modified Bank # (Note changes or attach parent)

New X e

Question History: Last NRC Exam Question Cognitive Level: Memory or Fundamental Knowledge Comprehension or Analysis X 10 CFR Part 55 Content: 55.41 3 55.43 Comments:

Page 61 of 203

ES-401 Sample Written Examination Form ES-401-5 Question Worksheet

',dExamination Outline Cross-reference: Level RO SRO Tier # 2 Group # 2 KIA # 215001 A4.03 Importance Rating 3.0 TIPS:Ability to manually operate and/or monitor in the control room: Isolation Valves.

Proposed Question: Common 31 The plant is operating at 90% power. The Transverse In-Core Probe(TIPS) is being used to recalibrate the LPRMs. A TIP probe is currently in the core when station blackout conditions occur.

At this time if a Group II isolation signal actuated the TIP probe would A. Not retract because the ball valve failed closed on the loss of power physically preventing TIP probe movement. The associated shear valve will automatically fire to isolate the system from containment.

B. Automatically retract. Because the ball valve failed open on the loss of power, the associated shear valve will automatically fire to isolate the system from co ntainment .

C. Not automatically retract. The associated shear valve must then be manually fired to isolate the system from containment.

D. Automatically retract. Because the ball valve failed open on the loss of power, the associated shear valve must then be fired manually to isolate the system from containment .

Proposed Answer: C Explanation (Optional):

A. Incorrect. Shear valve requires manual operation.

B. Incorrect. Shear valve requires manual operation.

C. Correct. Ball valve fails close on power loss. The TIP drive mech also loses power (same power supply). Shear valve can be manually fired using key lock switch.

D. Incorrect. Ball valve fails close on power loss.

Page 62 of 203

ES-401 Sample Written Examination Form ES-401-5 Question Worksheet Technical Reference@) . TIPS reference text, Revl, (Attach if not previously provided) sections C.8 and E.4.

Proposed references to be provided to applicants during examination: NONE Learning 0bjective: (As available)

Question Source: Bank #

Modified Bank # (Note changes or attach parent)

New X Question History: Last NRC Exam Question Cognitive Level: Memory or Fundamental Knowledge Comprehension or Analysis X 10 CFR Part 55 Content: 55.41 (7)

Comments:

Page 63 of 203

ES-401 Sample Written Examination Form ES-401-5 Question Worksheet ij Examination Outline Cross-reference: Level RO SRO Tier # 2 Group # 2 WA # 215002 A3.01 Importance Rating 3. I Ability to monitor automatic operations of the Rod Block Monitor System including: Four Rod Display.

Proposed Question: Common 32 The plant is starting up and all APRMs are currently reading 23% power.

While withdrawing a control rod from position 24 to 26, reed switch 25 failed to open.

The rod is presently latched at notch 26.

Indication(s) for this condition will be (1) and the Rod Block Monitor system

-(2)- generate a rod block signal.

A. (1) Red drift light on full core display and Black (no indication) for the rod on the four-rod display.

(2) Will

-L/

B. (1) Red drift light on full core display and both 25 and 26 superimposed on the four rod display.

(2) Will NOT.

C. (I) Red drift light on full core display and Black (no indication) for the rod on the four-rod display.

(2) Will NOT.

D. (7) Red drift light on full core display and both 25 and 26 superimposed on the four rod display.

(2) Will.

Page 64 of 203

ES-40 1 Sample Written Examination Form ES-401-5 Question Worksheet Proposed Answer: B

..J Explanation (Optional):

A. Incorrect. 25 and 26 will be superimposed, RBM not affected at <25.9% power.

B. Correct. A reed switch could fail such that it is either stuck open or stuck closed. If either an even or odd numbered switch is stuck closed, the position associated with that switch will be superimposed on the display along with the true position of the rod. The RBM channel is automatically bypassed if the reference APRM output is less than 25.9 percent power.

C. Incorrect. 25 and 26 will be superimposed.

D. Incorrect. RBM not affected at ~25.9%power.

Technical Reference(s) RPlS reference text, Rev.3, (Attach if not previously provided) page 14, section F.3.e.

RBM reference text, Rev.7, page 18,section 4 Proposed references to be provided to applicants during examination: NONE Learning Objective: (As available)

Question Source: Bank ##

L-'

Modified Bank # (Note changes or attach parent)

New X Question History: Last NRC Exam Question Cognitive Level: Memory or Fundamental Knowledge Comprehension or Analysis X 10 CFR Part 55 Content: 55.41 6 55.43 Comments:

Page 65 of 203

ES-401 Sample Written Examination Form ES-401-5 Question Worksheet

'L/ Examination Outline Cross-reference: Level RO SRO Tier # 2 Group # 2 WA # 216000 2.2.25 Importance Rating 2.5 Equipment control knowledge of b a s e s in tech s p e c s for LCOs and Safety Limits.

Proposed Question: Common 33 Which ONE (I 1 of the following describes the (1) Technical Specification bases for the Reactor Pressure (76 psig) or Reactor Level (-46") settings and (2) A reason why the specific setpoint was chosen?

A. (I) The Reactor Pressure setpoint bases is to protect RHR Shutdown Cooling piping from overpressurization.

(2) So that the shutdown cooling isolation valves can close in the required time.

B. (1) The Reactor Pressure setpoint bases is to protect RWCU reject piping from overpressurization.

(2) So that the maximum differential pressure associated with the isolation valves is not exceeded.

C. (1) The Reactor Level setpoint bases is used to activate the CSCS subsystems.

(2) So that it would be low enough to prevent actuation during failure to scram events.

D. (1) The Reactor Level setpoint bases is used to close all isolation valves.

(2) To be adequate to assure at least 213 core coverage in case of a break in the largest line.

Proposed Answer: A Explanation (Optional):

A. Correct. Per TS bases 3.2 the reactor low pressure was chosen to protect SD cooling piping from overpressurization and in part so that the isolation valves would close in the require time.

B. Incorrect. Not based on RWCU reject piping.

C Incorrect. Bases is prevent spurious actuation and core damage.

D. Incorrect. Does not close all isolation valves.

Page 66 of 203

ES-401 Sample Written Examination Form ES-401-5 Question Worksheet W Technical Reference(s) TS bases 3.2 protective (Attach if not previously provided) instrumentation Proposed references to be provided to applicants during examination: NONE Learning Objective: (As available)

Question Source: Bank #

Modified Bank # (Note changes or attach parent)

New X Question History: Last NRC Exam Question Cognitive Level: Memory or Fundamental Knowledge X Comprehension or Analysis 10 CFR Part 55 Content: 55.41 3 55.43 Comments:

\e Page 67 of 203

ES-401 Sample Written Examination Form ES-401-5 Question Worksheet ii Examination Outline Cross-reference: Level RO SRO Tier # 2 Group # 2 KIA # 226001 - K1 . I 1 Importance Rating 2.8 Knowledge of the physical relationships andlor cause/effect relationships between RHWLPCI: ContainmentSpray Mode and the following: CCW Proposed Question: Common 34 The plant is at rated conditions when the B RBCCW loop is lost due to an unisolable pipe break. When Containment parameters begin to degrade, the Rx is scrammed.

Future procedurally driven actions necessary to control containment parameters will include:

A. Torus Sprays and Vessel Depressurization.

B. Torus Sprays and Drywell Sprays.

C. Drywell Sprays and Vessel Depressurization.

D. Torus Cooling and Drywell Cooling.

Proposed Answer: A Explanation (Optional):

A. Correct. AS directed by EOP-03 given RBCCW loop Byis unavailable B. Incorrect. Parameters for DW spray initiation not met. (not a LOCA condition)

C. Incorrect. Parameters for DW spray initiation not met. (not a LOCA condition)

D. Incorrect. DW cooling not available with loss of RBCCW loop B.

Technical Reference@) EOP-03 (Attach if not previously provided)

PNPS 2.4.42 Loss of RBCCW Proposed references to be provided to applicants during examination: NONE Learning Objective: (As available)

Question Source: Bank #

Modified Bank # (Note changes or attach parent)

Page 68 of 203

ES-401 Sample Written Examination Form ES-401-5 Question Worksheet New X Question History: Last NRC Exam Question Cognitive Level: Memory or Fundamental Knowledge Comprehension or Analysis X 10 CFR Part 55 Content: 55.41 5 55.43 Comments:

Page 69 of 203

ES-401 Sample Written Examination Form ES-401-5 Question Worksheet w' Examination Outline Cross-reference: Level RO SRO Tier # 2 Group # 2 WA # 241000 A3.09 Importance Rating 3.3 Ability to monitor automatic operations of the W u r b . Pressure Regulator System including: ControllGovernor valve operation.

Proposed Question: Common 35 PNPS is shutting down and the plant is at 20% power. The turbine generator is synched to the grid. A grid disturbance results in a sudden large load drop causing a main turbine overspeed condition.

Assuming that the turbine does NOT trip. Which ONE (1) of the following can be observed in the control room that indicate the MHC system has taken automatic action to limit the overspeed condition?

A. ONLY the turbine control valves close.

B. ONLY the intercept valves close.

%..J' C. Both the turbine control valves and intercept valves close.

D. Both the turbine control valves and stop valves close.

Proposed Answer: C Explanation (Optional):

A. Incorrect. Intercepts also throttle closed.

B. Incorrect. Control valves also throttle closed.

C. Correct. If a load reject were to occur (at less than turbine stop valve closure and turbine control valve fast closure bypass setpoint of approx. 25% power), the acceleration relay senses the fast drop in speed signal caused by the speed increase, and rapidly closes the control valves and intercept valves.

D. Incorrect. Intercepts throttle closed not stops.

Technical Reference(s) MHC reference text, Rev. 3, (Attach if not previously provided) section 3.b, page 58 Page 70 of 203

ES-401 Sample Written Examination Form ES-401-5 Question Worksheet Proposed references to be provided to applicants during examination: NONE 4

Learning Objective: (As available)

Question Source: Bank # MHC-3932 Modified Bank # (Note changes or attach parent)

New Question History: Last NRC Exam Question Cognitive Level: Memory or Fundamental Knowledge X Comprehension or Analysis 10 CFR Part 55 Content: 55.41 3 55.43 Comments:

Page 71 of 203

~

ES-401 Sample Written Examination Form ES-401-5 Question Worksheet LJ Examination Outline Cross-reference: Level RO SRO Tier # 2 Group # 2 WA # 256000 A I .04 Importance Rating 2.9 Ability to predict and/or monitor changes in parameters associated with operating the reactor condensate system controls including:

hotwell level.

Proposed Question: Common 36 The plant is operating at 100% power.

You are currently lowering level in the hotwell using LIC-3300 to open fine reject valve LV-3300A. Coarse reject valve LV-3301B is currently in AUTO and CLOSED.

The following alarm is then received in the control room:

0 RFP N B I C SUCT PRESS LO, Panel C l L D-I Based on the alarm, how would your ability to continue hotwell reject with the coarse and fine reject valves, from the control room, be affected?

\4 A. Reject would still be possible but with the fine reject valve ONLY. The coarse reject valve is interlocked closed.

B. Reject would still be possible with both the coarse and fine reject valves.

C. Reject would still be possible but with the coarse reject valve ONLY. The fine reject valve auto closes.

D. Reject would not be possible because both the coarse and fine reject valves are interlocked closed.

Page 72 of 203

ES-401 Sample Written Examination Form ES-401-5 Question Worksheet Proposed Answer: D

.L.J Explanation (Optional):

A. Incorrect. Both valves receive auto close signal.

B. Incorrect. Both valves receive auto close signal.

C. Incorrect. Both valves receive auto close signal.

D. Correct. An overriding signal from the RFP is sent to shut the condensate reject valves and condensate minimum flow valve if RFP low NPSH is sensed. The alarm that came in is indicative of this condition.

Technical Reference(s) FW and Condensate (Attach if not previously provided)

Reference Text, Rev.5, page 36, section D.4.

Alarm Response: RFP A/B/C SUCT PRESS LO , Panel C1L D-I.

Proposed references to be provided to applicants during examination:

Learning Objective: (As available)

,- Question Source: Bank #

Modified Bank # (Note changes or attach parent) c-New X Question History: Last NRC Exam Question Cognitive Level: Memory or Fundamental Knowledge X Comprehension or Analysis 10 CFR Part 55 Content: 55.41 4 55.43 Comments:

Page 73 of 203

ES-401 Sample Written Examination Form ES-401-5 Question Worksheet L/ Examination Outline Cross-reference: Level RO SRO Tier # 2 Group # 2 KIA # 272000 K6.03 Importance Rating 2.8 Knowledge of the effect that a loss or malfunction of the following will have on the Rad monitoring system: AC power Proposed Question: Common 37 The plant is operating at 80% power.

All systems are operable with the exception of the Air Ejector Off-Gas LRM 1705-3A which has a locked in false Hi-Hi alarm. I&C is investigating the problem. The radiation monitoring system is aligned for normal operation.

Power is then lost to RPS channel B due to an EPA breaker trip.

These conditions will cause  ?

A. A half scram and a Carbon Bed Bypass isolation to occur.

c, B. A half scram and an isolation of off-gas after a 13 minute timer times out.

C. A half scram and an immediate isolation of off-gas.

D. A half scram ONLY.

Proposed Answer: B Explanation (Optional):

A. Incorrect. Only for the post treatment.

6. Correct. RPS B powers LRM 1705-36, If both radiation monitors are high-high, downscale or one channel hiqh-hiqh and the other one downscale (due to power loss from RPS supply) , a trip will be initiated. It will start the 13 minute timer from the monitors selected by the radiation monitor selector switch. After 13 minutes, the off-gas holdup volume will isolate. Loss of power to RPS initiates a half scram.

C. Incorrect. Off gas only isolates after a 13 minute timer times out.

D. Incorrect. Off gas only isolates after a 13 minute timer times out.

Page 74 of 203

ES-401 Sample Written Examination Form ES-401-5 Question Worksheet Technical Reference(s) PRM reference text, Rev. 5, (Attach if not previously provided)

-J pages 13 to 16, Alarms page 27 Proposed references to be provided to applicants during examination:

Learning Objective: (As available)

Question Source: Bank #

Modified Bank # (Note changes or attach parent)

New X Question History: Last NRC Exam Question Cognitive Level: Memory or Fundamental Knowledge Comprehension or Analysis X 10 CFR Part 55 Content: 55.41 7 55.43 Comments:

\u I Page 75 of 203

ES-401 Sample Written Examination Form ES-401-5 Question Worksheet W Examination Outline Cross-reference: Level RO SRO Tier # 2 Group # 2 K/A # 228000 A4.01 Importance Rating 3.1 Ability to operate and/or monitor in the control room: Plant ventilation -start and stop fans.

Proposed Question: Common 38 The plant is at 100% power and all equipment is operable.

The field operator calls the control room and reports that the both control room air conditioning units have tripped. The operator also reports that the supply and recirc/exhaust fans are operating normally.

As temperatures rise in the control room, the operators are required to perform the following in accordance with PNPS 2.4.149, LOSSof Control Room Air Conditioning.

A. Initiate the once through cooling mode of operation.

.e B. Initiate CRHEAF Supply Fan AOR CRHEAF Supply Fan 5.

C. Initiate both CRHEAF Supply Fan A AND CRHEAF Supply Fan B.

D. Place the standby supply and recirdexhaust fans in service.

Proposed Answer: A Explanation (Optional):

A. Correct. By procedure, if only the air conditioning is lost and the supply and exhaust fans are available then initiate the once through cooling mode of ventilation.

B. Incorrect. Only true if supply and exhaust fans are not available.

C. Incorrect. Only true if supply and exhaust fans are not available.

D. Incorrect. Not a procedure step.

Technical Reference(s) PNPS 2.4.1 49, Rev.I O , step (Attach if not previously provided) 4.0[3]and [5]

Page 76 of 203

ES-401 Sample Written Examination Form ES-401-5 Question Worksheet Proposed references to be provided to applicants during examination: NONE L.J Learning Objective: (As available)

Question Source: Bank #

Modified Bank # (Note changes or attach parent)

New X Question History: Last NRC Exam Question Cognitive Level: Memory or Fundamental Knowledge X Comprehension or Analysis 10 CFR Part 55 Content: 55.41 10 55.43 Comments:

Page 77 of 203

ES-401 Sample Written Examination Form ES-401-5 Question Worksheet iJ Examination Outline Cross-reference: Level RO SRO Tier # 1 Group # I WA # 295001 AA2.05 Importance Rating 3.1 Ability to determine andlor interpret the following as they apply to Partial or Complete loss of Forced Core Flow Circulation: Jet Pump Operability.

Proposed Question: Common 39 The plant is going to be placed in single loop operation due to a maintenance issue on the B reactor recirculation pump motor.

While performing the jet pump operability check and in determining total core flow, when the B Recirc pump is shutdown, you are required to account for indications of reverse flow through the inactive jet pump loop.

Which one of the following is consistent with reverse flow through the idle loop?

A. On Total Core Flow Recorder, FR-263-1I O , if, after the pump is shut down, Total Core Flow indication lowers and then rises.

IJ B. If the in-service pump speed is raised and idle jet pump flow lowers as a result, then reverse flow exists.

C. If the in-service pump speed is lowered and idle jet pump flow rises as a result, then reverse flow exists.

D. Flow indicators [Fl-263-107A (Fl-263-107B)l will indicate positive flow.

Proposed Answer: A Explanation (Optional):

A. Correct. Per reactor recirc reference text - On Total Core Flow Recorder, FR-263-1I O , if, after the pump is shut down, Total Core Flow decreases to a minimum and then increases B. incorrect. If pump speed and idle loop flow increase.

C. Incorrect. If pump speed and idle loop flow increase.

D. incorrect. This does not definitely indicate reverse core flow. Other parameters must be checked.

Page 78 of 203

ES-401 Sample Written Examination Form ES-401-5 Question Worksheet Technical Reference(s) Reactor Recirc Ref. Text. , (Attach if not previously provided)

L J Rev. 10, section C.7., page 25 Proposed references to be provided to applicants during examination: NONE Learning Objective: (As available)

Question Source: Bank #

Modified Bank # (Note changes or attach parent)

New X Question History: Last NRC Exam Question Cognitive Level: Memory or Fundamental Knowledge Comprehension or Analysis X 10 CFR Part 55 Content: 55.41 2, 3 55.43 Comments:

Page 79 of 203

ES-401 Sample Written Examination Form ES-401-5 Question Worksheet L--,,, Examination Outline Cross-reference: Level RO SRO Tier # 1 Group # 1 KIA # 295003 MI.03 Importance Rating 4.4 Ability to operate andlor monitor the following as they apply to Partial or complete loss of AC power: Systems necessary to assure a safe plant shutdown.

Proposed Question: Common 40 With grid voltage steadily lowering, the control room initiated a reactor scram. When the reactor was scrammed a loss of feed occurred.

Reactor water level dropped off the narrow range instruments about 4 minutes after the scram and stabilized at about -60 inches due to feed from RCIC.

0 15 minutes after the scram, reactor pressure was stable at 900 psig and controlled with SRVs.

0 Grid voltage stabilized, and A-1 thru A-4 were energized from the SUT with bus voltages of 3700 volts. A-5 and A-6 are on the EDGs at 4200 volts.

The A CRD pump has tripped and will not restart. This is due to ?

A. The CRD pump has been operating at near maximum flow since the scram, resulting in the pump overheating causing a breaker trip.

B. The CRD suction clogged due to operating at high flows causing the CRD pump to trip on low NPSH.

C. The prolonged low voltage condition has caused a low voltage trip of the CRD pump breaker.

D. A diesel generator load shed has occurred resulting in the CRD pump breaker being tripped and prevented from restarting.

Page 80 of 203

ES-401 Sample Written Examination Form ES-401-5 Question Worksheet Proposed Answer: D

'\ -1 Explanation (Optional):

A. Incorrect. Minimum flow would not trip pump and prevent restart.

B. Incorrect. No trip on low NPSH. Although a low suction pressure trip exists it would not prevent pump restart.

C. Incorrect. A low voltage condition did not cause the trip. The load shed did.

D. Correct. There are two bus degraded voltage relays in each emergency bus, 127A-A-5,l (2). If voltage drops to 3960 volts (3949 to 3978.29) for 10 seconds (9.88 to 10.60), these relays will actuate the bus undervoltage alarm ("A-5 and A-6 UNDERVOLTAGE" ON C3LC/A-3 AND C3-LCIB-3) in the Control Room. If a LOCA signal is present with the SUT feeder supplying the bus during this condition, a load shed will result. In this case level is at -60 inches which is below the -46 inch LOCA load shed setpoint thereby shedding the CRD pump breaker and preventing reclosure.

Technical Reference(s) Emergency AC dist. Ref text, (Attach if not previously provided)

Rev.7 Section C.7.

RDH reference text, Rev.4, section D.4. page 29 Proposed references to be provided to applicants during examination: NONE Learning Objective: (As available)

.,d Question Source: Bank # Emerg. AC

-#3934 Modified Bank # (Note changes or attach parent)

New Question History: Last NRC Exam Question Cognitive Level: Memory or Fundamental Knowledge Comprehension or Analysis X 10 CFR Part 55 Content: 55.41 7. I O 55.43 Comments:

Page 81 of 203

ES-401 Sample Written Examination Form ES-401-5 Question Worksheet L/ Examination Outline Cross-reference: Level RO SRO Tier # 1 1 Group # 1 1 KIA # 295004 AK2.03 Importance Rating 3.3 Knowledge of the interrelations between PARTIAL OR COMPLETE LOSS OF D.C. POWER and the following: D.C. bus loads Proposed Question: Common 41 The reactor is operating at 100% power when a loss of 125V DC bus D-5 occurs. The reactor is scrammed and actions are taken per PNPS 5.3.12. Loss of Essential DC Bus 0-17 or D-5 and D-37.

A large break LOCA then occurs resulting in the following conditions:

0 Drywell Pressure: 18 psig and rising 0 Reactor Level: -140 inches and lowering 0 Reactor Pressure: 100 psig and lowering How will the Core Spray system respond to these conditions?

d A. Only the A Core Spray pump will start and inject.

B. Only the B Core Spray pump will start and inject.

C. Both the Aand B Core Spray pumps will start and inject.

D. Both Core Spray pumps A and B will NOT start.

Proposed Answer: A Explanation (Optional):

A. Correct - Loss of D-5 only affects the control power for B core spray pump. A pump control power available from D-4.

B. Incorrect- Loss of D-5 removes power from the B core spray pump.

C. Incorrect - Loss of D-5 removes power from the B core spray pump.

D. Incorrect - Loss of D-5 removes power from the B core spray pump.

Page 82 of 203

ES-401 Sample Written Examination Form ES-401-5 Question Worksheet Technical Reference(s) DC Reference Text, Rev.6, (Attach if not previously provided)

'U' section 3.b.

Proposed references to be provided to applicants during examination: NONE Learning Objective: (As available)

Question Source: Bank #

Modified Bank# X (Note changes or attach parent)

New Question History: Last NRC Exam 8/2003 - modified to opposite train Question Cognitive Level: Memory or Fundamental Knowledge Comprehension or Analysis X 10 CFR Part 55 Content: 55.41 7 55.43 Comments:

12/1 - Gil - needs more references u

Page 83 of 203

ES-401 Sample Written Examination Form ES-401-5 Question Worksheet u' Examination Outline Cross-reference: Level RO SRO Tier # 1 Group # 1 WA # 295005 AKI .03 Importance Rating 3.5 Knowledge of the operational implications as they apply to the Main Turbine Generator Trip: Pressure affects on reactor level.

Proposed Question: Common 42 During the performance of PNPS 8.A.9-1, Turbine Test Weekly, at 90% reactor power, a malfunction occurs resulting in a main turbine trip and then a reactor scram.

Initially reactor pressure will -(I) resulting in indicated reactor vessel level to

-(2)-7 A. (I) Rise (2) Rise B. (1) Rise (2) Lower

-L/ C. (1)Lower (2) Rise D. (1) Lower (2) Lower Proposed Answer: B Explanation (Optional):

A. Incorrect. Misunderstanding plant response on turbine trip and where vessel level is measured could lead to wrong answer.

B. Correct. Increases until SRWbypass valves lift, pressure increase causes indicated level to decrease C. Incorrect. Misunderstanding plant response on turbine trip and where vessel level is measured could lead to wrong answer.

D. Incorrect. Misunderstanding plant response on turbine trip and where vessel level is measured could lead to wrong answer.

Technical Reference(s) Thermodynamic principles (Attach if not previously provided)

Page 84 of 203

ES-401 Sample Written Examination Form ES-401-5 Question Worksheet

' x - 1 Proposed references to be provided to applicants during examination: NONE Learning Objective: (As available)

Question Source: Bank #

Modified Bank # (Note changes or attach parent)

New X Question History: Last NRC Exam Question Cognitive Level: Memory or Fundamental Knowledge Comprehension or Analysis X 10 CFR Part 55 Content: 55.41 5 55.43 Comments:

Page 85 of 203

ES-401 Sample Written Examination Form ES-401-5 Question Worksheet J Examination Outline Cross-reference: Level RO SRO Tier # 1 Group # 1 K/A # 295006 AK2.05 Importance Rating 3.1 Knowledge of the interrelations between Scram and the following: CRD mechanism.

Proposed Question: Common 43 The plant has just scrammed due to low reactor vessel level signal. The Control Rod Drive HCUs ensure control rod insertion by allowing water to flow from the precharged accumulator through valve 126 to  ?

A. The CRDM underpiston area. The CRDM overpiston area is vented through valve 127 to the scram discharge volume.

6. The CRDM overpiston area. The CRDM underpiston area is vented through valve 127 to the scram discharge volume.

C. The CRDM overpiston area. The CRDM underpiston area is vented through C valve 127 to the exhaust header.

D. The CRDM underpiston area. The CRDM overpiston area is vented through valve I 2 7 to the exhaust header.

Proposed Answer: A Explanation (Optional):

A. Correct. When a scram signal is present, scram pilot solenoid valves (117 and 118) vent air pressure from the scram inlet and outlet valves (126 and 127) and cause them to open.

Water then flows from the precharged accumulator through valve 126 to the CRDM underpiston area. CRDM overpiston area is vented through valve 127 to the scram discharge volume.

B. Incorrect. Opposite of correct answer.

C. Incorrect. Water is vented to the scram discharge volume not the exhaust header.

D. Incorrect. Water is vented to the scram discharge volume not the exhaust header.

Technical Reference(s) CRDH reference text, Rev. 4, (Attach if not previously provided)

I page 8, section B.4 d

Page 86 of 203

ES-401 Sample Written Examination Form ES-401-5 Question Worksheet

.L/ Proposed references to be provided to applicants during examination: NONE Learning Objective: (As available)

Question Source: Bank #

Modified Bank # (Note changes or attach parent)

New X Question History: Last NRC Exam Question Cognitive Level: Memory or Fundamental Knowledge X Comprehension or Analysis I O CFR Part 55 Content: 55.41 7 55.43 Comments:

Page 87 of 203

ES-401 Sample Written Examination Form ES-401-5 Question Worksheet L/ Examination Outline Cross-reference: Level RO SRO Tier # 1 Group # 1 WA # 295016 AK3.03 Importance Rating 3.5 Knowledge for the reasons of the following responses as they apply to control room abandonment :disabling control room controls.

Proposed Question: Common 44 The control room must be abandoned due to a fire. The immediate actions of PNPS 2.4.143 have been taken and an operator is standing by the HPCl Alternate Shutdown Panel ASP C155.

In order to disable the control room controls and actuate the local panels the operators must perform several breaker manipulations and also place five control switches on ASP C155 in -(I)- . These actions will prevent all HPCl automatic functions except the -(2)-.

A. (1) Pull-to-Lock (2) HPCl Isolation B. (I) Pull-to-lock (2) HPCl Overspeed Trip C. (1)Normal (2) HPCl Isolation D. (?)Normal (2) HPCl Overspeed Trip Proposed Answer: D Explanation (Optional):

A. Incorrect. Switches must be placed in Normal, only overspeed functions B. Incorrect. Switches must be placed in normal .

C. Incorrect. Only overspeed functions.

D. Correct. Switches must be placed in Normal, only the overspeed trip functions all others are bypassed due to switches being in normal.

Page 88 of 203

ES-401 Sample Written Examination Form ES-401-5 Question Worksheet Technical Reference(s) 2.4.143, Rev.36, App. A, step (Attach if not previously provided) 2.0[2] and step 3.0[6]

Proposed references to be provided to applicants during examination: NONE Learning Objective: (As available)

Question Source: Bank #

Modified Bank # (Note changes or attach parent)

New X Question History: Last NRC Exam Question Cognitive Level: Memory or Fundamental Knowledge X Comprehension or Analysis 10 CFR Part 55 Content: 55.41 10 55.43 Comments:

Page 89 of 203

ES-401 Sample Written Examination Form ES-407-5 Question Worksheet

-eExamination Outline Cross-reference: Level RO SRO Tier # 1 Group # 1 WA # 295018 AK3.05 importance Rating 3.2 Knowledge of the reasons for the following responses as they apply to partial or complete loss of component cooling water: Placing standby Ht exchanger in service.

Proposed Question: Common 45 A major winter storm has hit the plant site resulting in the total loss of intake structure equipment.

The plant has been scrammed and offsite power is still available. Plant parameters are stable and in their normal band following the scram.

Which ONE (I) of the following is an approved method of maintaining cooling to plant loads that can be implemented after the scram per PNPS 5.3.3 Loss of all Service Water?

L d A. Place RHR Loop A in torus cooling and establish a Fire Water connection directly to the RBCCW side of the RHR Loop Aheat exchanger.

6. Place RHR Loop Aand RHR Loop B in torus cooling. RBCCW Loop Awill cool both loops of RHR.

C. Place RHR Loop Ain torus cooling and cross tie RBCCW Loops A and B.

D. Establish a Fire Water connection directly to the RBCCW side of the RHR Loop Bheat exchanger.

Page 90 of 203

ES-401 Sample Written Examination Form ES-401-5 Question Worksheet Proposed Answer: C

-1 Explanation (Optional):

A. Incorrect. These methods are not described in the procedure.

B. Incorrect. These methods are not described in the procedure.

C. Correct. Per PNPS PNPS 5.3.3 Loss of all Service Water this method has been approved as an immediate way to maintain drywell cooling using the Torus as heat sink for drywell heat loads.

D. Incorrect. These methods are not described in the procedure.

Technical Reference(s) PNPS PNPS 5.3.3Loss of all (Attach if not previously provided)

Service Water (0 to 3 hr steps) page 6.

Proposed references to be provided to applicants during examination:

Learning Objective: (As available)

Question Source: Bank #

Modified Bank # (Note changes or attach parent)

New X L.2 Question History: Last NRC Exam Question Cognitive Level: Memory or Fundamental Knowledge Comprehension or Analysis X 10 CFR Part 55 Content: 55.41 IO 55.43 Comments:

Page 91 of 203

ES-401 Sample Written Examination Form ES-401-5 Question Worksheet 4 Examination Outline Cross-reference: Level RO SRO Tier # I Group # 1 WA # 295019 AK3.023.5 Importance Rating Knowledge of the reasons for the following responses as they apply to partial or complete loss of instrument air: Standby air compressor operation.

Proposed Question: Common 46 The plant is operating at 100% power with all systems operable when the following alarm is received:

BACKUP DIESEL COMPRESSOR RUNNING - C2R-D4 This alarm indicates that the air header has decreased to -(I)- and that the Backup Diesel Compressor will continue to run until -(2)-.

A. (I) 102 psig (2) It is manually shutdown.

-ul B. (1) 102 psig (2) It auto shuts down at 110 psig.

C. (I) 90 psig (2) It is manually shutdown.

D. ( I ) 90 psig (2) It auto shuts down at 110 psig.

Proposed Answer: C Explanation (Optional):

A. Incorrect. The Backup Diesel compressor gets a start signal at 90 psig. (102 is the start for the lead rotary air compressor)

6. Incorrect. The backup diesel compressor gets a start signal at 90 psig.

C. Correct. The backup diesel compressor starts at 90 psig and must be manually secured.

D. Incorrect. It must be manually secured.

d Technical Reference(s) PNPS 5.3.8, Rev.29, section (Attach if not previously provided)

Page 92 of 203

ES-401 Sample Written Examination Form ES-401-5 Question Worksheet 2.0 Proposed references to be provided to applicants during examination: NONE Learning Objective: (As available)

Question Source: Bank #

Modified Bank # (Note changes or attach parent)

New X Question History: Last NRC Exam Question Cognitive Level: Memory or Fundamental Knowledge X Comprehension or Analysis 10 CFR Part 55 Content: 55.41 4 55.43 Comments:

Page 93 of 203

ES-401 Sample Written Examination Form ES-401-5 Question Worksheet

--./

Examination Outline Cross-reference: Level RO SRO Tier # 1 Group # I WA # 295002 2.4.49 Importance Rating 4.0 Ability to perform without reference to proceduresthose actions that require immediate operation of system components and controls.

Proposed Question: Common 47 The plant is at 70% power and Core Flow at 50 mlbm/hr The control room has just received the following alarms:

AFTER CNDSR LOOP SEAL LVL HI/LO - CP600L-A8 COOLER CNDSR LOOP SEAL LVL LO - CP6OOL- B8 You then notice condenser vacuum begin to degrade.

What are the immediate actions you must take in accordance with PNPS 2.4.36 Decreasing Condenser Vacuum if the situation continues to degrade?

A. Reduce reactor power using the RPR array to stop the vacuum from degrading.

At 22 inches Hg, scram the reactor then trip the Main Turbine.

6. Reduce reactor power with the recirculation pumps to stop the vacuum from degrading. At 22 inches Hg, scram the reactor.

C. Reduce reactor power using the RPR array to stop the vacuum from degrading.

Before vacuum degrades to 22 inches Hg, trip the Main Turbine then scram the reactor.

D. Reduce reactor power using reactor recirculation pumps to stop the vacuum from degrading. Before vacuum degrades to 22 inches Hg, scram the reactor.

Page 94 of 203

ES-401 Sample Written Examination Form ES-401-5 Question Worksheet Proposed Answer: D LJ Explanation (Optional):

A. Incorrect. By procedure Power must be reduced with recirc not rods. 22 is the turbine trip setpoint.

B. Incorrect. 22 inches is the turbine trip setpoint.

C. Incorrect, Guidance is to scram the reactor before the turbine trips.

D. Correct. Immediate actions of PNPS 2.4.36 for decreasing vacuum are to reduce reactor power with recirc in an attempt to improve vacuum. If vacuum is approaching 22 inches HG and cant be recovered, scram the reactor.

Technical Reference(s) PNPS 2.4.36, Rev.35, section (Attach if not previously provided) 9 Proposed references to be provided to applicants during examination: NONE Learning Objective: (As available)

Question Source: Bank #

Modified Bank # (Note changes or attach parent)

New X b

Question History: Last NRC Exam Question Cognitive Level: Memory or Fundamental Knowledge X Comprehension or Analysis 10 CFR Part 55 Content: 55.41 4, 10 55.43 Comments:

Page 95 of 203

ES-401 Sample Written Examination Form ES-401-5 Question Worksheet Ll Examination Outline Cross-reference: Level RO SRO Tier # 1 Group # 1 WA # 295021 AKI .03 Importance Rating 3.9 Knowledge of the operational implications as they apply to Loss of shutdown cooling: adequate core cooling.

Proposed Question: Common 48 The plant is cooling down and Shutdown Cooling is in service. Reactor pressure is currently 50 psig and the Main Condenser is not available. A loss of power then occurs to the running RHR pump. Attempts to restart any RHR pump are unsuccessful.

In order to assure adequate decay heat removal, which one of the methods shall be used in accordance with PNPS 2.4.25 Loss of Shutdown Cooling.

A. Place RWCU in service and cross tie RBCCW loops.

9. Increase reactor vessel level to at least +60 inches.

e C. Restart a Reactor Recirculation Pump D. Defeat the low pressure RCIC isolation and place RCIC in service.

Proposed Answer: B Explanation (Optional):

A. Incorrect. No procedural guidance for these actions with the reactor still pressurized and the condenser available.

B. Correct. Without the main condenser available you raise level to promote natural circulation.

C. Incorrect. No procedural guidance for these actions with the reactor still pressurized and the condenser available.

D. Incorrect. Not procedurally driven for this scenario.

Technical Reference(s) PNPS 2.4.25,Rev.29, Loss of (Attach if not previously provided)

SD Cooling, 4.0[6].

Page 96 of 203

ES-401 Sample Written Examination Form ES-401-5 Question Worksheet Proposed references to be provided to applicants during examination: NONE

  • -J Learning Objective: (As available)

Question Source: Bank #

Modified Bank # (Note changes or attach parent)

New X Question History: Last NRC Exam Question Cognitive Level: Memory or Fundamental Knowledge Comprehension or Analysis X 10 CFR Part 55 Content: 55.41 IO 55.43 Comments:

Page 97 of 203

ES-401 Sample Written Examination Form ES-401-5 Question Worksheet d Examination Outline Cross-reference: Level RO SRO Tier # 1 Group # 1 KIA # 295032 EK 2.01 Importance Rating 3.4 Knowledge of the interrelations between high secondary containment temperature and: Arealroom coolers.

Proposed Question: Common 49 Following a scram and loss of off-site power, HPCl is placed in pressure control mode. A small HPCl steam leak then results in slowly rising HPCl area temperatures.

Which one of the following describes the response of the HPCl area coolers to the rising area temperature?

Assume all equipment to support HPCl operation was in the normal configuration prior to placing HPCI in service.

A. VAC-20lA auto started when the HPCI Steam Admission Valve opened upon system startup. Standby area cooler, VAC-201B, will auto start when area temperature rises to 105 degrees.

B. VAC-201A will auto start when area temperature rises to 95 degrees. Standby area cooler,

--=

VAC-2016, will auto start if low flow is sensed in VAC-201A.

C. VAC-201A will auto started when area temperature rises to 95 degrees. Area cooler, VAC-201B, will auto start when area temperature rises to 105 degrees.

D. VAC-201A and VAC-201B will not auto start until area temperature rises to the Max Normal Operating Value but can be manually started by placing their control switches to the RUN position.

Proposed Answer: C Explanation (Optional):

A is incorrect because area coolers only auto start on high area temperature B is incorrect because the B cooler auto starts on hi area temperature and not low flow C is the correct response D is incorrect because the coolers can only be manually started by taking the control switches to TEST position Technical Reference(s) PNPS 2.2.47, page 8 (Attach if not previously provided)

Page 98 of 203

ES-401 Sample Written Examination Form ES-401-5 Question Worksheet Proposed references to be provided to applicants during examination: NONE Learning Objective: (As available)

Question Source: Bank #

Modified Bank # (Note changes or attach parent)

New X Question History: Last NRC Exam Question Cognitive Level: Memory or Fundamental Knowledge X Comprehension or Analysis 10 CFR Part 55 Content: 55.41 10, 1I 55.43 Comments:

Page 99 of 203

ES-401 Sample Written Examination Form ES-401-5 Question Worksheet Examination OutI ine Cross-reference: Level RO SRO Tier # 1 Group # 1 WA # 295024 EA1.I 2 Importance Rating 3.8 High Drywell Pressure: Ability to operate andlor monitor the following as they apply to High drywell Pressure: Suppression Pool Spray.

Proposed Question: Common 50 The Low Pressure Coolant injection (LPCI) system has just initiated and the following plant conditions exist:

Reactor vessel water level is (-50 inches).

0 Drywell Pressure is 1.5 psig.

Which of the following is/are required to allow operation of the torus spray valves MO-I 001-37A(B),?

(1) Drywell Pressure rises to ~ 1 . psig 8 (2) Take the LPCl Override switch to MANUAL OVERIDE u'

(3) Take the RPV Level Override keylock switch to MANUAL OVERIDE A. ONLY (1).

B. ONLY (2).

C. ONLY (1) and (2).

D. ONLY (2) and (3).

Page 100 of 203

ES-40 I Sample Written Examination Form ES-401-5 Question Worksheet Proposed Answer: C d Explanation (Optional):

D. Incorrect. (2) also required.

D. Incorrect. (1) also required.

D. Correct. If the LPCl initiation white light is on, you must rotate the knurled knob (S17A(B)-

LPCl OVERIDE) to the right to override the LPCl initiation auto closure for the containment spray/coolingvalves. Even if the LPCl signal is overridden, there must be

->I .8 psig drywell pressure before the 37s can be opened if a LPCI initiation signal is present.

D. Incorrect. (I) is required, (3) is not required.

Technical Reference(s) RHWLPCI reference text, (Attach if not previously provided)

Rev.7, sections 9.e.6 (page 40) and 11.b.2. (page 44)

Proposed references to be provided to applicants during examination: NONE Learning Objective: (As available)

Question Source: Bank # RHWLPCI-8648 d

Modified Bank # (Note changes or attach parent)

New Question History: Last NRC Exam Question Cognitive Level: Memory or Fundamental Knowledge X Comprehension or Analysis 10 CFR Part 55 Content: 55.41 7 55.43 Comments:

Page 101 of 203

ES-401 Sample Written Examination Form ES-401-5 Question Worksheet 4 Examination Outline Cross-reference: Level RO SRO Tier # I Group # 1 KIA # 295025 EKI .01 Importance Rating 3.9 Knowledge of the operational implications of the following concepts as they apply to high reactor pressure: pressure effects on reactor power.

Proposed Question: Common 51 Following a Main Turbine trip, reactor pressure peaked at I180 psig and reactor water level lowered to -60 inches and is slowly recovering. While responding to the transient you report that both Reactor Recirculation pumps are running.

The Control room supervisor directs you to trip both pumps because the high reactor pressure and low-low level condition should have caused -(I)-? Tripping the pumps will -(2)-.

A. (1) ONLY the Recirc Pump generator field breakers to trip.

(2) Add negative reactivity to counteract the positive reactivity addition caused by the initial pressure spike.

B. (1) ONLY the Recirc Pump drive motor breakers to trip.

(2) Add negative reactivity to counteract the positive reactivity addition caused by the pressure decrease following the scram.

C. (I) Each Recirc Pump drive motor breaker AND generator field breaker to trip.

(2) Add negative reactivity to counteract the positive reactivity addition caused by the initial pressure spike.

D. (1) Each Recirc Pump drive motor breaker AND generator field breaker to trip.

(2) Add negative reactivity to counteract the positive reactivity addition caused by the pressure decrease following the scram.

Page 102 of 203

ES-401 Sample Written Examination Form ES-401-5 Question Worksheet Proposed Answer: C 4 Explanation (Optional):

A. Incorrect. Drive motor breakers also trip.

6. Incorrect. Generator field breakers also trip.

C. Correct. Both breakers trip, pump trip adds - reactivity to counteract positive reactivity from pressure spike.

D. Incorrect. Pressure decrease does not add positive reactivity.

Technical Reference(s) PNPS 2.2.84, Rev.90, steps (Attach if not previously provided) 4.3.3 and 4.3.4 Proposed references to be provided to applicants during examination: NONE Learning Objective: (As available)

Question Source: Bank #

Modified Bank # (Note changes or attach parent)

New X Question History: Last NRC Exam L d Question Cognitive Level: Memory or Fundamental Knowledge Comprehension or Analysis X 10 CFR Part 55 Content: 55.41 3, 5 55.43 Comments:

Page 103 of 203

ES-40I Sample Written Examination Form ES-401-5 Question Worksheet 4 Examination 0utIine Cross-reference: Level RO SRO Tier # 1 Group # 1 WA # 295026 EA1.03 Importance Rating 3.9 Ability to operate and/or monitor the following as they apply to suppression pool high water temperature: Temperature monitoring.

Proposed Question: Common 52 Which ONE (1) of the following describe the requirements for the monitoring of Suppression Chamber Tem perature.

A. During the performance of a HPCl surveillance test, suppression chamber temperatures must be continuously monitored and logged every 5 minutes until Torus temperatures have stabilized after heat addition has been terminated.

B. During the performance of a RClC surveillance test, suppression chamber temperatures must be continuously monitored and logged every 5 minutes until temperatures are below the technical specification limit of 90 degrees F.

u' C. During the performance of a HPCl surveillance test, suppression chamber temperatures must be continuously monitored and logged every 10 minutes until Torus temperatures have stabilized after heat addition has been terminated.

D. During the performance of a RClC surveillance test, suppression chamber temperatures must be continuously monitored and logged every 10 minutes until temperatures are below the technical specification limit of 80 degrees F.

Proposed Answer: A Explanation (Optional):

A. Correct. PNPS 2.1 . I 9 "Suppression Chamber Temperatures" step 8.0[3].

B. Incorrect. Not in accordance with 2.1.19.

C. incorrect. Not in accordance with 2.1.19.

D. incorrect. Not in accordance with 2.1.19.

Page 104 of 203

ES-401 Sample Written Examination Form ES-401-5 Question Worksheet Technical Reference(s) PNPS 2.1.I 9 Suppression (Attach if not previously provided) w Chamber Temperatures Proposed references to be provided to applicants during examination:

Learning Objective: (As available)

Question Source: Bank #

Modified Bank # (Note changes or attach parent)

New X Question History: Last NRC Exam Question Cognitive Level: Memory or Fundamental Knowledge X Comprehension or Analysis 10 CFR Part 55 Content: 55.41 10 55.43 Comments:

Page 105 of 203

ES-401 Sample Written Examination Form ES-401-5 Question Worksheet L-.,, Examination Outline Cross-reference: Level RO SRO Tier # 1 Group # 1 KIA # 295028 EA2.03 Importance Rating 3.7 Ability to determine and/or interpret the following as they apply to High Drywell Temperature: Reactor Water Level.

Proposed Question: Common 53 During accident conditions with elevated drywell temperatures, the more accurate vessel level indication are found on -(I)-because -(2)-.

A. (1) Narrow Range Level Indicators (2) Most of the vertical portions of the reference legs are outside the drywell.

B. (I) Feedwater Level Indicators (2) Most of the horizontal portions of the reference legs are outside the drywell C. (1) Feedwater Level Indicators (2) Most of the vertical portions of the reference legs are outside the drywell.

u D. (1) Narrow Range Level Indicators (2) Most of the horizontal portions of the reference legs are outside the drywell Proposed Answer: A Explanation (Optional):

A. Correct.

E. Incorrect.

C. Incorrect.

D. Incorrect.

Technical Reference(s) NBI Reference text, Rev.5, (Attach if not previously provided) pg.8 Proposed references to be provided to applicants during examination: NONE Learning Objective: (As available)

Question Source: Bank #

Modified Bank # (Note changes or attach parent)

Page 106 of 203

ES-401 Sample Written Examination Form ES-401-5 Question Worksheet New X

'.+'

Question History: Last NRC Exam Question Cognitive Level: Memory or Fundamental Knowledge X Comprehension or Analysis 10 CFR Part 55 Content: 55.41 2 55.43 Comments:

Page 107 of 203

pESAO1 ~ Sample Written Examination Form ES-401-5 Question Worksheet u Examination Outline Cross-reference: Level RO SRO Tier # I Group # 1 KIA # 295030 EK3.01 Importance Rating 3.8 Knowledge of the reasons for the following responses as they apply to Low Suppression Pool Water Level: Emergency Depressurization.

Proposed Question: Common 54 An unisolable leak has occurred in the bottom of the Torus.

The plant is scrammed and EOPs are entered.

EOP-3 directs Emergency Depressurization (ED) when Torus level cannot be maintained above -(I)- inches to ensure that-(2)-?

A. (1)95 (2) Opening an SRV will not result in exceeding the allowable design stresses in SRV tailpipes.

v B. (1)90 (2) Prevent pressurization of the Torus airspace by HPCI steam exhaust.

c. (1)95 (2) Prevent pressurization of the Torus airspace by RClC steam exhaust.

D. (1)90 (2) Minimize the impact of a loss of pressure suppression capability.

Proposed Answer: D Explanation (Optional):

A. Incorrect. That would correspond to 175 inches, EOP-03, step TL-7.

B. Incorrect. That would correspond to 95 inches, EOP-03, step TL-13.

C. incorrect. RCIC is not a concern since exhaust flow is about equal to decay heat.

D. Correct. EOP-03, step TL-15.

Page 108 of 203

ES-401 Sample Written Examination Form ES-40 1-5 Question Worksheet Technical Reference(s) EOP-03, Rev. 8 (Attach if not previously provided)

Proposed references to be provided to applicants during examination: EOPs (no bases or entry conditions)

Learning Objective: (As available}

Question Source: Bank #

Modified Bank # (Note changes or attach parent)

New X Question History: Last NRC Exam Question Cognitive Level: Memory or Fundamental Knowledge X Comprehension or Analysis 10 CFR Part 55 Content: 55.41 2, 5 55.43 Comments:

Page 109 of 203

ES-401 Sample Written Examination Form ES-401-5 Question Worksheet u Examination Outline Cross-reference: Level RO SRO Tier # I Group # 1 WA # 295031 2.2.2 Importance Rating 3.4 Rx Low Water Level: Equipment Control Knowledge of bases in TS for LCOs for operations and safety limits.

Proposed Question: Common 55 Due to transmitter failures, the Reactor Low Water Level primary containment isolation system (PCIS) instruments are below the Technical Specification (TS) minimum requirement for operable instruments in each trip system.

The associated PClS TS LCO actions for Reactor Low Water Level (+12inches) are based on .

A. Ensuring that adequate level is maintained so that post accident cooling can be accomplished and the guidelines of 10 CFR IO0 would not be violated.

B. Ensuring that adequate level exists to prevent core uncovery in the case of a l- l break of the largest line assuming a 60 second valve closing time.

C. Ensuring the low pressure core thermal power safety limit is not exceeded due to fuel uncovery.

D. Ensuring the MCPR safety limit is not exceeded.

Proposed Answer: B Explanation (Optional):

A. Incorrect. Per TS bases 3.2 this would be for Low-Low level.

B. Correct. Per TS bases 3.2.

C. Incorrect. Per TS bases 3.2 this would be MSlV closure.

D. Incorrect. Per TS bases 3.2.

Technical Reference(s) TS bases 3.2 (Attach II not previously provided)

~~~~ ~

+ Proposed references to be provided to applicants during examination: TS - no bases Page 110 of 203

ES-401 Sample Written Examination Form ES-401-5 Question Worksheet

\ .-. ,. Learning Objective: (As available)

Question Source: Bank #

Modified Bank # (Note changes or attach parent)

New X Question History: Last NRC Exam Question Cognitive Level: Memory or Fundamental Knowledge X Comprehension or Analysis 10 CFR Part 55 Content: 55.41 2, 5, 7 55.43 Comments:

Page 111 of 203

ES-401 Sample Written Examination Form ES-401-5 Question Worksheet

-J Examination Outline Cross-reference
Level RO SRO Tier # 1 Group # 1 K/A # 295037 2.4.6 Importance Rating 3.1 Emerg. ProcedureslPlan Knowledge based EOP Mitigation Strategies (scram condition present and power > aprm dowrnscale or unknown Proposed Question: Common 56 The following conditions exist in the plant:

0 Boron is being injected into the vessel with the SLC system.

The initial SLC tank level was 4300 gallons.

0 Reactor water level band was intentionally lowered.

0 Reactor water level band is -100 to -135.

Torus water temperature is 1 15 degrees F.

Current SLC tank level is 3400 gallons.

Which ONE (I) of the following action(s) islare required in accordance with the EOPs?

A. Emergency RPV Depressurization is required Per EOP-02, Then raise reactor water level to a band of +12 to +45 Per EOP-01.

6. Raise reactor water level to a band of 12 to 45Per EOP-02.

C. Continue to lower reactor level to -150, Then Emergency RPV Depressurize Per EOP-02.

D. Maintain reactor water level in its present band until Cold Shutdown Boron Weight has been achieved Per EOP-02, then enter EOP-01.

Page 112 of 203

ES-401 Sample Written Examination Form ES-401-5 Question Worksheet Proposed Answer: B e Explanation (Optional):

A. Incorrect. ED is not required per EOP-02.

B. Correct. Hot Shutdown Boron Weight has been achieved, therefore per EOP-02, steps L-7 and L-23, level can be restored to +12 to +45.

C. Incorrect. ED is not required per EOP-02.

D. Incorrect. Not an action in EOP-02.

Technical Reference(s) EOP-02 (Attach if not previously provided)

Proposed references to be provided to applicants during examination: EOPs - no entry conditions Learning Objective: (As available)

Question Source: Bank #

Modified Bank # (Note changes or attach parent)

New X Question History: Last NRC Exam L x Question Cognitive Level: Memory or Fundamental Knowledge Comprehension or Analysis X 10 CFR Part 55 Content: 55.41 7, 10 55.43 Comments:

Page 113 of 203

ES-401 Sample Written Examination Form ES-401-5 Question Worksheet

.j Examination Outline Cross-reference: Level RO SRO Tier # I Group # I KIA # WA 295038 EA1.01 Importance Rating 3.9 4.2 Ability to operate and/or monitor the following as they apply to high off-site release rate: Stack Gas monitoring system.

Proposed Question: Common 57 Following a fuel element failure, the reactor is manually scrammed and a cooldown is commenced. Standby Gas is verified to have responded as expected. Operators note that the low range Main Stack Rad Monitors (PRM-1705-18A & B) begin to rise and then stabilize approximately 4 times their normal full power reading.

Subsequently, the running Stack Dilution Fan trips and the standby Dilution Fan fails to start.

Following the reduction in stack dilution flow, Main Stack Rad Monitor readings (PRM-1705-18A & B) will (1) and Actual /total Off site release rate will (2)

A. ( I ) rise (2) rise B. (1)lower (2) remain the same C. (1) lower (2) lower D. ( I ) rise (2) remain the same Proposed Answer: D Explanation (Optional):

A. A decrease in stack flow increases release concentrations although total releases is unaffected. Since PRMs respond to concentrations,a decrease in stack flow will cause the PRMs to indicate higher concentrations. Ais therefore incorrect since total release does not change.

B. PRM reading would increase C. PRM reading would increase and total release would remain the same.

D. Correct response Technical Reference(s) PNPS 2.4.45, page 5,third (Attach if not previously provided)

Page 114 of 203

ES-401 Sample Written Examination Form ES-401-5 Question Worksheet paragraph Proposed references to be provided to applicants during examination: NONE Learning Objective: (As available)

Question Source: Bank #

Modified Bank # (Note changes or attach parent)

New X Question History: Last NRC Exam Question Cognitive Level: Memory or Fundamental Knowledge X Comprehension or Analysis 10 CFR Part 55 Content: 55.41 IO, 12 55.43 Comments:

12/1 -Gil - suggested SBGT system operability issue

.'v-I 12/27 - steve - question written but is a weak link to KA.

Page 115 of 203

ES-401 Sample Written Examination Form ES-401-5 Question Worksheet Examination Outline Cross-reference: Level RO SRO Tier # 1 Group # 1 WA # 600000 AA1.06 Importance Rating 3.0 Ability to operate and/or monitor the following as they apply to plant fire on site: fire alarm system.

Proposed Question: Common 58 A control room alarm indicates that a detector actuation in one smoke detector zone of the Cable Spreading Room (CSR) Halon system has occurred.

Based on this alarm a Halon actuation -(I)- occurred.

If required, the Halon system can be manually activated -(2)-.

A. (1) Has.

(2) Remotely outside each access door to the CSR.

i j

6. ( I ) Has Not.

(2) Remotely outside each access door to the CSR.

C. (?)Has (2) Remotely from the Control Room.

D. (1) Has Not.

(2) Remotely from the Control Room.

Proposed Answer: B Explanation (Optional):

A. incorrect. Has not.

B. Correct. The system is activated by a trip of both of its smoke detection zones OR locally at the Halon cylinders OR remotely outside each access door to the CSR C. Incorrect. No control from control room.

D. Incorrect. No control from control room.

Page 116 of 203

ES-401 Sample Written Examination Form ES-401-5 Question Worksheet Technical Reference(s) per PNPS 2.2.129, Rev.11 (Attach if not previously provided)

Halon System steps 4.2.1 and 4.3 Proposed references to be provided to applicants during examination: NONE Learning Objective: (As available)

Question Source: Bank # Fire prot.

System

  1. 5349 Modified Bank # (Note changes or attach parent)

New Question History: Last NRC Exam Question Cognitive Level: Memory or Fundamental Knowledge X comprehension or Analysis 10 CFR Part 55 Content: 55.41 4, 10 55.43 v

Comments:

Page I17 of 203

ES-401 Sample Written Examination Form ES-401-5 Question Worksheet

  • L/

Examination Outline Cross-reference: Level RO SRO Tier # 1 Group # 2 WA # 295009 AK2.03 Importance Rating 3.1 Knowledge of the interrelations between low reactor water level and the recirc system.

Proposed Question: Common 59 The following plant conditions exist:

The B and C Reactor Feed Pumps are in service.

0 Reactor Power is at 75%.

A failure then occurs on the Feedwater Regulating valves reducing feedwater flow to 3.3 Mlb/hr. Reactor vessel level reaches + I 6 inches before operators take manual control of the feedwater regulating valves and return level to +30 inches. .

No action was taken on the Recirculation system.

Which ONE (I) of the following describes the response of the Recirculation Flow

-L.J Controllers?

A. The speed demand signal goes to 26% not subject to rate limiting.

B. The speed demand signal goes to 26% subject to a 1.5% per second rate limit.

C. The speed demand signal goes to 44% not subject to rate limiting.

D. The speed demand signal goes to 44% subject to a 1.5% per second rate limit.

Page I18 of 203

ES-401 Sample Written Examination Form ES-401-5 Question Worksheet Proposed Answer: C

\d Explanation (Optional):

A. incorrect. Limiter goes to 44%.

B. Incorrect. No rate limit on limiter #2 (44%).

C. Correct. When reactor level is ,19 and less than 3 feed pumps are in service speed limiter #2 overides the controller ouput and limits speed to 44% with no rate limit.

D. Incorrect. No rate limit on limiter #2 (44%).

Technical Reference(s) PNPS 2.2.84, Rev. 90, step (Attach if not previously provided) 4.2.4[2](6) and (e)

Proposed references to be provided to applicants during examination: NONE Learning Objective: (As available)

Question Source: Bank # Recirc sys.

  1. 3265 Modified Bank # (Note changes or attach parent)

New

.L-.,, Question History: Last NRC Exam Question Cognitive Level: Memory or Fundamental Knowledge Comprehension or Analysis X 10 CFR Part 55 Content: 55.41 5, 7 55.43 Comments:

Page 119 of 203

ES-401 Sample Written Examination Form ES-401-5 Question Worksheet

.J Examination Outline Cross-reference: Level RO SRO Tier # I Group # 2 WA # 29501 3 AA1.01 Importance Rating 3.9 Ability to operate or monitor the following as it applies to High Suppression Pool temperature: suppression pool cooling.

Proposed Question: Common 60 A loss of coolant accident has occurred. The following Plant conditions exist:

0 Reactor Water Level is -1 60 inches.,

0 Drywell Pressure is 2.5 psig Torus Temperature is 1I O degrees F.

You are directed to place Torus Cooling in service.

Which ONE (I) of the following describe the actions, if any, required to be taken to open Torus Cooling Valves MO-34NB and MO-36 N B .

-J A. No actions, valves can be opened because reactor level did not lower to below 2/3 core Coverage.

B. Place RPV Level Override Keylock switch to MANUAL OVERRIDE.

C. Momentarily place the LPCl Override switch to MANUAL OVERRIDE.

D. Place RPV Level Override Keylock switch to MANUAL OVERRIDE AND momentarily place the LPCl Override switch to MANUAL OVERRIDE .

Page 120 of 203

ES-401 Sample Written Examination Form ES-401-5 Question Worksheet Proposed Answer: D u

Explanation (Optional):

A. Incorrect. LPCl override switch must be moved.

B. Incorrect. Both switches must be moved.

C. Incorrect. Both switches must be moved.

D. Correct. MO-23A(B), 26A(B), 34A(B), 36A(B) and 37A(B) are shut during LPCl initiation indicated by a white light above containment spray permissive reset switch. To reopen these valves, the containment spray valve control switch S-I 7 must be placed in MANUAL for loop A(B). Valves MO-34 and 36 can be operated. You must have 1.8 psig drywell pressure in order to operate MO-23, 26 and 37. A white light, located above the manual override keylock switch, indicates that the interlock is bypassed.

Technical Reference(s) RHR reference Text, Rev. 7, (Attach if not previously provided) page 57 Proposed references to be provided to applicants during examination:

Learning Objective: (As available)

Question Source: Bank #

Modified Bank # (Note changes or attach parent)

New X Question History: Last NRC Exam Question Cognitive Level: Memory or Fundamental Knowledge Comprehension or Analysis X IO CFR Part 55 Content: 55.41 7, 10 55.43 Comments:

Page 121 of 203

ES-401 Sample Written Examination Form ES-401-5 Question Worksheet

\d Examination Outline Cross-reference: Level RO SRO Tier # 1 Group # 2 KIA # 295014 AK2.01 Importance Rating 3.9 Knowledge of the interrelations between Inadvertent Reactivity Addition and RPS.

Proposed Question: Common 61 The plant is starting up following a refuel outage. Reactor Power is in the SRM/IRM overlap range. All systems are operable.

An uncoupled control rod near the center of the core then drops to position 48.

How would RPS respond to this condition?

A. With all shorting links (red, blue, green and yellow) installed, any NI instrument trip would result in a full reactor scram.

B. With only the red and blue shorting links removed, IRMs and APRMs are in W

coincidence logic and a half scram would occur.

C. With all shorting links (red, blue, green and yellow) removed, any NI instrument trip will cause a full reactor scram.

D. With only the red and blue shorting links removed, ONLY SRMs are in non-coincidence logic and a half scram would occur.

Page 122 of 203

ES-401 Sample Written Examination Form ES-401-5 Question Worksheet Proposed Answer: C

'd Explanation (Optional):

A. Incorrect. Shorting links must be removed for non coincident logic.

B. Incorrect. All shorting links must be removed (not just the red and blue).

C. Correct. In some instances, sensor contact arrangements remove coincidence requirements of trip systems and permit any one of a number of sensors to initiate a full reactor scram. This is the case for non-coincidence NMS logic. Normally installed red and blue shorting links may be removed from manual channel logics to permit any one NMS auxiliary relay (K13 A, B, C, or D) to trip manual scram channels. Removal of yellow and green shorting links from SRM sensor contacts enables SRM "hi-hi" trips to de-energize corresponding K13 relays in addition to IRM and APRM trips. Thus with all shortina links (red, blue, green and yellow) removed, any NMS instrument trip will cause a full reactor scram. With only red and blue shorting links removed, IRMs and APRMs are in non-coincidence logic and SRMs are still entirely bypassed providing no protective trips.

D. Incorrect. In this situation they are in non-coincidence logic.

Technical Reference(s) RPS reference text, Rev.4, (Attach if not previously provided) section B.14 Proposed references to be provided to applicants during examination: NONE

'W' Learning Objective: (As available)

Question Source: Bank #

Modified Bank # (Note changes or attach parent)

New X Question History: Last NRC Exam Question Cognitive Level: Memory or Fundamental Knowledge Comprehension or Analysis X 10 CFR Part 55 Content: 55.41 55.43 Comments:

Page 123 of 203

ES-401 Sample Written Examination Form ES-401-5 Question Worksheet

~LI Examination Outline Cross-reference: Level RO SRO Tier # 1 Group # 2 WA ## 295015 AA2.02 Importance Rating 4.1 Ability to interpret and/or determine the following as it applies to incomplete scram: control rod position.

Proposed Question: Common 62 EOP-01 is entered following a Main Turbine trip.

Which ONE (1) of the following would indicate that an entry into EOP-02 is required?

A. 20 control rod positions indicate 04,all other control rod positions indicate 00 with the green FULL IN lights on.

6.With the exception of one control rod at position 36, all other control rod positions indicate 00 with the green FULL INlights on.

C. With the exception of one control rod at position 06 all other control rod

-.---, positions indicate 02 .

D. All control rod positions indicate 00 with the green FULL IN lights NOT illuminated.

Proposed Answer: C Explanation (Optional):

A. Incorrect. This meets the Max Subcritical Banked Withdrawal Position (MSBWP) definition of all rods to 04 or beyond.

8. Incorrect. Meets TS 4.3.A.1 definition all rods to 00 with the exception of one are acceptable per EOP-01 description of the step R-4 question Can existing rod pattern I always assure reactor shutdown.

C. Correct. This does not meet TS or MSBWP requirements.

D. Incorrect. If all rods indicate00 the full in lights need not be illuminated.

Technical Reference(s) EOP-01 ref. Text, TS 4.3.A. 1 (Attach if not previously provided)

PNPS 5.3.35, Rev. 9, pg 9 &I2 Page 124 of 203

ES-401 Sample Written Examination Form ES-401-5 Question Worksheet Proposed references to be provided to applicants during examination: EOPs - no entry conditions or bases Learning Objective: (As available)

Question Source: Bank #

Modified Bank # (Note changes or attach parent)

New X Question History: Last NRC Exam Question Cognitive Level: Memory or Fundamental Knowledge x Comprehension or Analysis 10 CFR Part 55 Content: 55.41 7 55.43 Comments:

Page 125 of 203

ES-401 Sample Written Examination Form ES-401-5 Question Worksheet u Examination Outline Cross-reference: Level RO SRO Tier # 1 Group # 2 WA # 295020 AK3.03 Importance Rating 3.2 Knowledge of the reasons for the following responses as they apply to Inadvertent Containment Isolation: DWlContainment temperature response.

Proposed Question: Common 63 An inadvertent Group Iisolation has occurred with the plant at 100% power. All other systems are operable. The reactor is successfully scrammed and reactor level and pressure are being controlled using EOP-01 and EOP-03 guidance.

0 Reactor Pressure - 800 psig decreasing 0 Reactor Level - (+lo Inches) increasing 0 Torus Level - 130 inches increasing 0 Torus Temperature - 90 degrees increasing 0 Drywell Pressure - 1.6 psig stable What will be the response of containment temperature to this event?

'v A. Drywell temperature will initially increase due the Group Iisolation causing a

. loss of drywell coolers. Drywell coolers can be restored when the isolation is reset.

B. Torus water temperature will increase due to the use of SRVs and/or HPCl to control reactor pressure. Suppression pool cooling must be placed in service to control increasing temperatures.

C. Drywell temperature will initially increase due to the loss of RBCCW to the drywell coolers. If the load shed is defeated the drywell coolers can be returned to service regardless of the Group 1 isolation.

D. Torus airspace temperature will increase due to the loss of drywell cooling.

Drywell coolers can be restored when the isolation is reset.

Page 126 of 203

ES-401 Sample Written Examination Form ES-401-5 Question Worksheet Proposed Answer: B u' Explanation (Optional):

A. Incorrect. DW coolers do not isolate on Group Iisolation.

B. Correct. A group 1 isolation causes the closure of the MSlVs and the condense will be unavailable as a heat sink. Therefore, HPCI, RCIC, SRVs will be controlling RPV pressure increasing suppression pool and torus air space temperatures.

C. Incorrect. No load shed signal occurred (No LOCA or LOP indications were given in stem).

D. Incorrect. No isolation occurred (No LOCA or LOP indications were given in stem).

Technical Reference(s) Primary Cont. Cooling System (Attach if not previously provided)

Reference Text, Rev.2, section C.l and 2.

PNPS 2.2.92, Rev. 51, section 4.2 Proposed references to be provided to applicants during examination: NONE Learning Objective: (As available)

Question Source: Bank #

Modified Bank # (Note changes or attach parent)

New X Question History: Last NRC Exam Question Cognitive Level: Memory or Fundamental Knowledge Comprehension or Analysis X 10 CFR Part 55 Content: 55.41 5,7 55.43 Comments:

Page 127 of 203

ES-401 Sample Written Examination Form ES-401-5 Question Worksheet

.L../ Examination Outline Cross-reference: Level RO SRO Tier ## 1 I Group # 2 2 WA # 295022 AK3.01 Importance Rating 3.7 Knowledge of the reasons for the following responses as they apply to LOSS OF CRD PUMPS: Reactor Scram.

Proposed Question: Common 64 The plant is operating at starting up and reactor pressure is currently 900 psig when the B CRD pump breaker trips. When the operators attempt to start the A CRD it fails to start.

The CRS directs you to closely monitor CRD accumulator trouble alarms because you are required to perform which one of the following in accordance with PNPS 2.4.4 Loss of CRD Pumps.

A. Scram the reactor when two or more accumulator trouble lights occur on the full core display regardless of the associated control rod position.

u 6. Immediately Scram the reactor if the charging water header pressure cannot be restored within 20 minutes.

C. Scram the reactor when two or more accumulator trouble lights occur on the full core display and the control rods cannot be verified fully inserted immediately.

D. Scram the reactor when two or more accumulator trouble lights occur on the full core display and the charging water header pressure cannot be restored within 20 minutes.

Proposed Answer: C Explanation (Optional):

A. Incorrect. Not per procedure guidance for the given conditions.

B. Incorrect. Not per procedure guidance for the given conditions.

C. Correct. Per immediate operator actions of PNPS 2.4.4for the initial conditions given.

D. Incorrect. Not per procedure guidance for the given conditions.

Technical Reference(s) PNPS 2.4.4,Rev. 20,section (Attach if not previously provided)

Page I 2 8 of 203

ES-401 Sample Written Examination Form ES-401-5 Question Worksheet 3.0 Proposed references to be provided to applicants during examination: NONE Learning 0bjective: (As available)

Question Source: Bank # #60 Modified Bank # (Note changes or attach parent)

New Question History: Last NRC Exam Question Cognitive Level: Memory or Fundamental Knowledge X Comprehension or Analysis 10 CFR Part 55 Content: 55.41 10 55.43 Comments:

12/1 - Gil- not written per procedure guidance 12/19 - Steve - revised

-../-,

Page 129 of 203

ES-401 Sample Written Examination Form ES-401-5 Question Worksheet v' Examination Outline Cross-reference: Level RO SRO Tier # 1 Group # 2 WA # 295029 EKI . O l importance Rating 3.4 Knowledge of the operational implications of the following as they apply to High Suppression Pool Water level: containment integrity, Proposed Question: Common 65 Regarding the primary containment systems response during a loss of coolant accident in the drywell:

Which one of the following failures or conditions could result in exceeding the NEGATIVE design pressure rating of containment?

A. Torus to Drywell vacuum Breaker failing open.

B. SRV tailpipe vacuum breaker failing closed.

C. Torus level rising to 175 inches with torus sprays in service.

D. Torus level rising to 190 inches with drywell sprays in service.

Proposed Answer: D Explanation (Optional):

A Incorrect. Torus to DW vacuum breakers failing open would threaten the design positive pressure limit.

B Incorrect. SRV vacuum breaker failing closed would increase level in the SRV tailpipe threatening tailpipe integrity and not negative pressure in dw.

C Incorrect. Torus spray header is covered at 300 inches.

D Correct. Torus to drywell vacuum breakers begin to cover preventing them from relieving to the drywell to prevent the drywell from going negative.

Technical Reference(s) EOP-03 Primary containment , (Attach if not previously provided)

Training material, Rev. 8 IG-14.

Proposed references to be provided to applicants during examination: NONE Learning Objective: (As available)

Page 130 of 203

ES-401 Sample Written Examination Form ES-401-5 Question Worksheet

'-.> Question Source: Bank #

Modified Bank # (Note changes or attach parent)

New X Question History: Last NRC Exam Question Cognitive Level: Memory or Fundamental Knowledge Comprehension or Analysis X 10 CFR Part 55 Content: 55.41 5, 10 55.43 Comments:

Page 131 of 203

ES-401 Sample Written Examination Form ES-401-5 Question Worksheet

\.-./

I Examination Outline Cross-reference: Level RO SRO Tier # 3 Group #

KIA # 2.1.18 Importance Rating 2.9 Ability to make clear, accurate and concise logs, records, status boards, and reports.

Proposed Question: Common 66 Which ONE (1) of the following describes the correct use of narrative logs as described in PNPS 1.3.34, Operations Administrative Policies and Procedures.

A. ONLY the Shift Manager can make changes to an approved eSOMS Narrative Log.

B. All operators on the shift may edit their entries on a turned over log.

C. The offgoing shift will complete the plant parameters for the next shift.

u D. The Shift Manager or the individual who made the entry can make changes to an approved eSOMS narrative log.

Proposed Answer: C Explanation (Optional):

A. Incorrect. No changes are permitted by anyone to an approved log.

B. Incorrect. Only the SM can edit a turned over log.

C. Correct. Per PNPS 1.3.34, step 6.5[1 I ]

D. Incorrect. Changes are permitted by anyone to an approved log.

Technical Reference(s) PNPS 1.3.34, Operations (Attach if not previously provided)

Administrative Policies and Procedure, Rev.111, section 6.5[4],[8],[1 I ]

Proposed references to be provided to applicants during examination: NONE Page 132 of 203

ES-401 Sample Written Examination Form ES-401-5 Question Worksheet "W' Learning Objective: (As available)

Question Source: Bank #

Modified Bank # (Note changes or attach parent)

New X Question History: Last NRC Exam Question Cognitive Level: Memory or Fundamental Knowledge X Comprehension or Analysis 10 CFR Part 55 Content: 55.41 10 55.43 Comments:

Page 133 of 203

ES-401 Sample Written Examination Form ES-401-5 Question Worksheet X d Examination Outline Cross-reference: Level RO SRO Tier # 3 Group #

WA # 2.1.8 Importance Rating 3.8 Ability to coordinate personnel activities outside the control room.

Proposed Question: Common 67 The control room must be evacuated due to habitability concerns. You are assigning the field duties for shift personnel. In accordance with PNPS 2.4.143, which ONE (1) of the following actions are permitted to be performed by non-licensed operators.

A. HPCl operation from the associated ASP B. RClC operation from the associated ASP.

C. SSW operation from the associated ASP.

D. SRV operation at the associated ASP.

i_/

Proposed Answer: C Explanation (Optional):

A. Incorrect. Can only be petformed by licensed operators.

B. Incorrect. Can only be performed by licensed operators.

C. Correct. Per PNPS 2.4.143 step 4.0[9](9)(e)(2).

D. Incorrect. Can only be performed by licensed operators.

Technical Reference@) PNPS 2.4.143, Rev.36, (Attach if not previously provided) shutdown from outside the control room sect. 4.0[9](e)(2)

Proposed references to be provided to applicants during examination: NONE Learning Objective: (As available)

Question Source: Bank #

b Page 134 of 203

ES-401 Sample Written Examination Form ES-401-5 Question Worksheet Modified Bank # (Note changes or attach parent)

New X Question History: Last NRC Exam Question Cognitive Level: Memory or Fundamental Knowledge X Comprehension or Analysis 10 CFR Part 55 Content: 55.41 10 55.43 Comments:

Page 135 of 203

ES-401 Sample Written Examination Form ES-401-5 Question Worksheet LJ' Examination Outline Cross-reference: Level RO SRO Tier # 3 Group #

WA # G2.1.25 Importance Rating 2.8 Ability to obtain and interpret station reference materials such as graphs, monographs and tables which contain performance data.

Proposed Question: Common 68 The control room has been evacuated due to fire in the control room. Shutdown from outside the control room is in progress per PNPS 2.4.143. A cooldown with SRVs has been initiated. The following data is taken at the indicated times.

TIME Rx PRESSURE RX PRESSURE 0o:oo 1000 psig 0050 600 psig 00: 10 920 psig 01 :oo 520 psig 00:20 840 psig 0l:lO 440 psig 00:30 760 psig 01:20 360 psig 00:40 680 psig 01:30 280 psig

.u' Which ONE (1) of the following groups of statements accurately describes the status of the cooldown rate.

A. The cooldown rate over the last hour averaged less than 90 degrees per hour.

The cooldown rate for the last 10 minutes averaged less than 90 degrees per hour.

B. The cooldown rate over the last hour averaged greater than 90 degrees per hour.

The cooldown rate for the last I O minutes averaged greater than 90 degrees per hour.

C. The cooldown rate over the last hour averaged less than 90 degrees per hour.

The cooldown rate for the last 10 minutes averaged greater than 90 degrees per hour.

D. The cooldown rate over the last hour averaged greater than 90 degrees per hour.

The cooldown rate for the last 10 minutes averaged less than 90 degrees per hour.

Page 136 of 203

ES-401 Sample Written Examination Form ES-401-5 Question Worksheet Proposed Answer: B

'.d Explanation (Optional):

0o:oo 1000 psig - 544 Deg. F. 00:50 600 psig - 486 00: I O 920psig -535 01:oo 520 psig - 471 00:20 840psig -524 01:lO 440 psig - 454 00:30 760 psig - 512 01:20 360 psig - 434 00:40 680 psig - 500 01:30 280 psig - 41 1 A. Incorrect. No other time period exceeded rate specified in question.

B. Correct. Per steam table (must convert psia to psig) 00:30 to 01:30 = 101 degrees >90 01:20 to 01:30 = 23 degrees X 6 =138, >90 C. Incorrect. No other time period exceeded rate specified in question.

D. Incorrect. No other time period exceeded rate specified in question.

Technical Reference(s) Steam Tables (Attach if not previously provided)

Proposed references to be provided to applicants during examination: Steam Tables Learning Objective: (As available)

-u Question Source: Bank # PNPS 5746 Modified Bank # (Note changes or attach parent)

New Question History: Last NRC Exam Question Cognitive Level: Memory or Fundamental Knowledge X Comprehension or Analysis 10 CFR Part 55 Content: 55.41 IO 55.43 Cornments :

Page I 3 7 of 203

W ES-401 Sample Written Examination Form ES-401-5 Question Worksheet

- Examination Outline Cross-reference: Level RO SRO Tier # 3 3 Group #

WA # G2.2:27 Importance Rating 2.6 Knowledge of the refueling process.

Proposed Question: Common 70 While moving a fuel bundle from the reactor vessel to the spent fuel pool, the electrical power supply is lost to the main hoist motor. The fuel bundle is presently hanging one foot above the top of the fuel storage rack.

Under these conditions:

A. The main hoist motor will automatically swap to its temporary power supply to lower the bundle.

B. The main hoist motor must be manually swapped to its temporary power supply to lower the bundle.

4 C. The main hoist motor will be without power and must be operated with a handwheel to lower the bundle.

D. The main hoist motor brakes will be without power and the bundle must be left in its current location until power is restored.

Proposed Answer: C Explanation (Optional):

A. Incorrect. The main hoist has only one power supply.

6. Incorrect. The main hoist has only one power supply.

C. Correct. Per PNPS 2.2.75, section 7.6 the main hoist has one power supply, the handwheel can be used to lower the bundle.

D. Incorrect. The motor will be without power.

Page 140 of 203

ES-401 Sample Written Examination Form ES-401-5 Question Worksheet Technical Reference(s) PNPS 2.2.75 Fuel Handling (Attach if not previously provided)

--..#/ and Service Equipment, Rev.45, section 7.6 and Section 7. I .4.

Refueling reference text, Rev.4, page 21 section 3.c.

Proposed references to be provided to applicants during examination:

Learning Objective: (As available)

Question Source: Bank # Fuel handling

  1. 2324 Modified Bank # (Note changes or attach parent)

New Question History: Last NRC Exam 11/2003 Question Cognitive Level: Memory or Fundamental Knowledge Comprehension or Analysis X I O CFR Part 55 Content: 55.41 5 55.43 Comments:

Gil- change distractor wording to temporary from alternate. Add word brake to distractor D

12/19 - Steve - updated references and revised Page 141 of 203

ES-401 Sample Written Examination Form ES-401-5 Question Worksheet

'i/ Examination 0ut1ine Cross-reference: Level RO SRO Tier # 3 Group #

WA # 2.2.34 Importance Rating 2.8 Knowledge of the process for determining the internal and external effects on core reactivity.

Proposed Question: Common 71 The plant is at 70% power and all equipment is operable. The following alarm is then received in the control room.

"IST POINT HTR LVL HI" (CIC-AI)

At that point you also observe that the extraction steam valve to the IST Point Heater E-105A has closed (MO-3156). Additionally on the EPIC Computer you notice that feedwater temperatures have trended down 10 degrees over the last 2 minutes.

Which ONE ( I ) of the following actions are you now required to take and why is that action taken?

.-' A. Reduce reactor power to a maximum of 45%. This action is required because a decrease in feedwater heating increases core inlet enthalpy resulting in an increase in core thermal power. Additional power reduction may be required to ensure the LHGR TS LCO is not exceeded.

6. Reduce reactor power to a maximum of 45%. This action is required because a decrease in feedwater heating reduces core inlet enthalpy resulting in an increase in core thermal power. Additional power reduction may be required to ensure the MCPR TS LCO is not exceeded.

C. Reduce reactor power to a maximum of 50%. This action is required because a decrease in feedwater heating increases core inlet enthalpy resulting in an increase in core thermal power. Additional power reduction may be required to ensure the LHGR TS LCO is not exceeded.

D. Reduce reactor power to a maximum of 50%. This action is required because a decrease in feedwater heating reduces core inlet enthalpy resulting in an increase in core thermal power. Additional power reduction may be required to ensure the MCPR TS LCO is not exceeded.

"L" Page 142 of 203

ES-401 Sample Written Examination Form ES-401-5 Question Worksheet

?

Proposed Answer: B

\J Explanation (Optional):

A. Incorrect. Loss of feedwater heating REDUCES core inlet enthalpy. Additional power reduction would not be necessary to ensure LHGR limits were met.

B. Correct. Per PNPS 2.4.1 50 Loss of feedwater heating - immediate actions must be at least a 25 YOreduction due to a reduction in core inlet enthalpy. Power may need further reduction to ensure MCPR limit is met C. Incorrect. Power must be reduced to a least 45% per procedure.

D. Incorrect. Power must be reduced to a least 45% per procedure.

Technical Reference(s) PNPS 2.4.1 50 Loss of (Attach if not previously provided) feedwater heating, Rev.20, immediate actions and sect.

5.0[4],[5].

Proposed references to be provided to applicants during examination: NONE Learning Objective: (As available)

Question Source: Bank #

v Modified Bank # (Note changes or attach parent)

New X Question History: Last NRC Exam Question Cognitive Level: Memory or Fundamental Knowledge Comprehension or Analysis X 10 CFR Part 55 Content: 55.41 5. 10 55.43 Comments:

12/22 - Gil suggested loss of nnl heating.

Page 143 of 203

ES-401 Sample Written Examination Form ES-401-5 Question Worksheet Examination Outline Cross-reference: Level RO SRO Tier # 3 Group #

WA # 2.3.1 1 Importance Rating 2.7 Ability to control rad releases.

Proposed Question: Common 72 Following the onset of Core Thermal Hydraulic Instabilities, the reactor is scrammed. The following plant conditions exist:

0 RPV level is being maintained 20 - 40 inches utilizing RClC 0 RPV pressure is 500 pounds and lowering utilizing SRVs Torus bottom pressure is 8 pounds and lowering utilizing torus sprays 0 Torus temperature is 104 degrees and rising RHR is aligned for maximum torus cooling utilizing RHR Pumps A and B 0 RHR pumps C and Dare in standby 0 The Shift Manager has just upgraded the event to a General Emergency due to elevated Torus CHARM readings With these initial conditions, an RHR A QUAD LEAKAGE (C904L - D7) alarm is received.

Torus level is observed to be slowly lowering. A field operator reports that there is a large leak on the suction of the A RHR pump and that quad level is currently 8 inches and rising.

Which one of the following actions are required by EOP-04, Secondary Control?

A. Commence pumping the Aquad using all available floor sump pumps.

Secure RHR pump Aand isolate the pump suction from panel C903.

B. Commence pumping the Aquad using all available floor sump pumps.

Maintain maximized torus cooling for as long as possible utilizing the A RHR pump.

Emergency Depressurize the reactor C. Open the breakers for the reactor building floor and equipment sump pumps Secure RHR pump A and isolate the pump suction from panel C903.

D. Open the breakers for the reactor building floor and equipment sump pumps Maintain maximized torus cooling for as long as possible utilizing the A RHR pump.

Emergency Depressurize the reactor Proposed Answer: C Explanation (Optional): The thrust of this question is to test the candidates ability to recognize that a core damaging event has occurred and that action must be taken to prevent the pumping of highly radioactive water outside the secondary containment via the floor sump pumps. This step was added to the PNPS EOPs following a review of the TMI event.

Page 144 of 203

ES-401 Sample Written Examination Form ES-401-5 Question Worksheet ij A. Incorrect. Indications of a severe core damaging event are present (Torus CHARMs >

General Emergency). Floor and Equipment sump pumps are to be disabled per EOP-04 step SC-1, 4h over-ride.

B. Incorrect. Indications of a severe core damaging event are present (Torus CHARMs >

General Emergency). Floor and Equipment sump pumps are to be disabled per EOP-04 step SC-1, 4thover-ride.

C. Correct response D. Incorrect response. RHR pump C can be placed in service to maintain torus cooling, allowing for the securing of pump A. Additionally, the RHR leak is not from a primary system and therefore Emergency Depressurizationis not required.

Technical Reference(s) EOP-043 Secondary (Attach if not previously provided)

Containment Control, step SC-1, 4hover-ride PNPS 5.3.35, page 15, step

[43](definition of Substantial Core Damage)

Proposed references to be provided to applicants during examination: NONE Learning Objective: (As available)

Question Source: Bank #

Modified Bank # (Note changes or attach parent)

New X Question History: Last NRC Exam Question Cognitive Level: Memory or Fundamental Knowledge Comprehension or Analysis X 10 CFR Part 55 Content: 55.41 I O , 11 55.43 Comments:

Page 145 of 203

ES-401 Sample Written Examination Form ES-401-5 Question Worksheet I

.d Examination Outline Cross-reference: Level RO SRO Tier # 3 Group #

WA # 2.3.1 Importance Rating 2.6 Knowledge of 10 CFR 20 and related facility radiation control requirements.

Proposed Question: Common 73 A Planned Special Exposure (PSE) is required.

A PSE is -(I)-.

When determining a PSE, 10 CFR 20 -(2)- are taken into account.

A. (1) Used whenever immediate measures are required to save lives or equipment.

(2) Annual TEDE limits of 10 Rem and Lifetime TEDE limits of 50 Rem.

B. (I) An infrequent exposure to radiation separate from and in addition to annual limits.

(2) Annual TEDE limits of 5 Rem and Lifetime TEDE limits of 25 Rem.

C. (1) Used whenever immediate measures are required to save lives or equipment.

(2) Annual TEDE limits of 5 Rem and Lifetime TEDE limits of 25 Rem.

D. (1) An infrequent exposure to radiation separate from and in addition to annual limits.

(2) Annual TEDE limits of I O Rem and Lifetime TEDE limits of 50 Rem.

Proposed Answer: B Explanation (Optional):

A. Incorrect. IOCFR 20 limits are 5R annual.

B. Correct. Per procedure definition of PSE and I O CFR 20 regulations.

C. Incorrect. Wrong definition of PSE.

D. Incorrect. Wrong 10 CRF 20 limit for annual TEDE.

Page 146 of 203

ES-401 Sample Written Examination Form ES-401-5 Question Worksheet Technical Reference(s) EN-RP-207, Rev.0, step3.0[1 I] (Attach if not previously provided) and 5.0[5]

10CFR20 Proposed references to be provided to applicants during examination: NONE Learning Objective: (As available)

Question Source: Bank #

Modified Bank # (Note changes or attach parent)

New X Question History: Last NRC Exam Question Cognitive Level: Memory or Fundamental Knowledge X Comprehension or Analysis 10 CFR Part 55 Content: 55.41 I O . 12 55.43 Comments:

Page 147 of 203

ES-401 Sample Written Examination Form ES-401-5 Question Worksheet

<.d Examination Outline Cross-reference: Level RO SRO Tier # 3 Group #

KIA # G2.4.5 Importance Rating 2.9 Knowledge of the organization of the operating procedures network for normal, abnormal, and emergency evolutions.

Proposed Question: Common 74 The plant is currently at 75% power and continuing a power ascension following a refueling outage.

While placing the 3 Reactor Feed Pump in service a reactor vessel level perturbation occurs causing reactor vessel level to decrease to 11 inches.

The operators are recovering reactor vessel level which is now at 25 inches and slowly rising.

Reactor power remains at approximately 75%. All other plant parameters are normal.

Based upon the current plant conditions the operators  ?

.u A. Shall immediately enter EOP-01, RPV Control and EOP-02, RPV Control Failure to Scram concurrently.

B. Shall immediately enter EOP-01, RPV Control.

C. Shall immediately commence an orderly reactor shutdown in accordance with PNPS 2.1.5 Controlled Shutdown From Power.

D. Shall immediately enter EOP-02, RPV Control Failure to Scram Page 148 of 203

ES-401 Sample Written Examination Form ES-401-5 Question Worksheet Proposed Answer: B

-4 Explanation (Optional):

A. Incorrect. Per EOP-01, EOP-02 is not entered unless a scram has been initiated failure to scram occurred (step r-4).

B. Correct. 12 vessel level is an EOP-01 entry condition.

C. Incorrect. Must enter EOP-01.

D. Incorrect. Must enter EOP-01 first. Entry to EOP-02 is from EOP-01.

Technical Reference(s) EOP-01, EOP-02, (Attach if not previously provided)

PNPS 5.3.35, Rev.9, pg 17 Proposed references to be provided to applicants during examination: EOPs without entry conditions or bases Learning Objective: (As available)

Question Source: Bank #

Modified Bank # (Note changes or attach parent)

New X Question History: Last NRC Exam L-.l Question Cognitive Level: Memory or Fundamental Knowledge Comprehension or Analysis X 10 CFR Part 55 Content: 55.41 10 55.43 Comments:

Page 149 of 203

ES-401 Sample Written Examination Form ES-401-5 Question Worksheet 4 Examination Outline Cross-reference: Level RO SRO Tier # 3 Group #

WA # 2.4.2 Importance Rating 3.9 Knowledge of system setpoints, interlocks and automatic actions assodated with EOP entry conditions.

Proposed Question: Common 75 The plant was operating at 100% power when the operators scrammed the reactor due to unexplained increasing drywell pressure. The reactor was scrammed but power is at 4%. Drywell cooling was lost and has not been restored.

Current plant conditions are:

0 Reactor Power at 4%

0 Reactor Pressure 500 psig 0 Drywell Temperature 140 degrees F. and increasing Drywell Pressure 2.3 psig and increasing Reactor Water level initially dropped to -22 inches but has recovered to +20 inches In addition to EOP-01; (I) The EOP entry conditions which were met due to this event.

(2) The reason for the automatic plant response of the RHR, Core Spray, and 4160 VAC systems.

A. (1) EOP-02 was entered as due to reactor water level low.

EOP-03was entered due to drywell temperature high.

(2) Low reactor water level initiated the start all RHR and Core Spray pumps and the EDGs.

B. ( I ) EOP-02 was entered as directed from EOP-01 due to an A W S condition.

EOP-03 was entered due to drywell pressure high.

(2) High Drywell Pressure initiated the start all RHR and Core Spray pumps and EDGs.

C. (I)EOP-02 was entered as directed from EOP-01 due to reactor water level low.

EOP-03 was entered due to drywell pressure high (2) Low reactor water level initiated the start all RHR and Core Spray pumps and the EDGs.

Page 150 of 203

ES-401 Sample Written Examination Form ES-401-5 Question Worksheet W D. (1) EOP-02 was entered as directed fromEOP-01 due to an ATWS condition.

EOP-03 was entered due to drywell temperature high.

(2) High Drywell Pressure initiated the start all RHR and Core Spray pumps and the EDGs.

Proposed Answer: B Explanation (Optional):

A. Incorrect. EOP-02 is not entered due to reactor water level low, EOP was not entered due to DW temp high (not high enough). Hi DW press. initiated equipment starts.

6. Correct. Per EOPs and equipment auto start signals.

C. Incorrect. EOP-02 is not entered due to reactor water level low, low water level did not initiate equipment starts.

D. Incorrect. EOP was not entered due to DW temp high (not high enough).

Technical Reference(s) EOPs 01, 02, 03 (Attach if not previously provided)

Proposed references to be provided to applicants during examination: EOPs without entry conditions "u*Learning Objective: (As available)

Question Source: Bank #

Modified Bank # (Note changes or attach parent)

New X Question History: Last NRC Exam Question Cognitive Level: Memory or Fundamental Knowledge Comprehension or Analysis X 10 CFR Part 55 Content: 55.41 7, 10 55.43 Comments:

Page 151 of 203

ES-401 Sample Written Examination Form ES-401-5 I Question Worksheet Examination Outline Cross-reference: Level RO SRO u Tier # 1 Group # 1 K/A # 295001 Importance Rating Ability to execute procedure steps Proposed Question: SRO 76 The reactor is operating at 100% when the A Reactor Recirculation pump trips. One minute later the BReactor Recirculation pump experiences a runback to the #2 speed limiter, Currently reactor power is 48% and core flow is 28 mlbmlhr.

Based on the power to flow map. Which ONE (I} of the following actions are you permitted to direct in accordance with PNPS 2.4.1 7 Recirc Pump Trip?

A. You are in the exclusion region. Restart the AReactor Recirc pump or insert control rods.

9. You are in the buffer zone. Immediately insert control rods or increase flow on the B Reactor Recirc pump.

4 C. You are in the exclusion region. Immediately insert control rods.

0. You are in the buffer zone. With SOLOMON as the stability monitor, exit the buffer zone if core or hot channel decay ratios exceed their limits.

Proposed Answer: C Explanation (Optional):

A incorrect. Not permitted to start a recirc pump to exit the exclusion region.

B Incorrect. Given conditions place the plant in the exclusion region not the buffer zone.

C Correct. The given conditions place the plant in the exclusion region. Per PNPS 2.4.17 for a recirc pump trip, if in the exclusion region refer to PNPS 2.1.14section 7.10 for changing power. Per the 2.1.14 guidance you must insert rods to exit the region.

Restarting a recirc pump is not acceptable.

D Incorrect. Given conditions place the plant in the exclusion region not the buffer zone.

Technical Reference(s) PNPS 2.4.17, Rev.37, Recirc (Attach if not previously provided)

Pump(s) Trip step 3, caution I.

PNPS 2.1 .I 4,Rev.91, Station Page 1 of 51

ES-401 Sample Written Examination Form ES-401-5 Question Worksheet Power Changes step 7.10 Proposed references to be provided to applicants during examination: Power to flow maps -

two loop and single loop.

Learning Objective: (As available)

Question Source: Bank #

Modified Bank # (Note changes or attach parent)

New X Question History: Last NRC Exam Question Cognitive Level: Memory or Fundamental Knowledge Comprehension or Analysis X 10 CFR Part 55 Content: 55.41 55.43 5 Comments:

J Page 2 of 51

ES-401 Sample Written Examination Form ES-401-5 Question Worksheet I

W Examination Outline Cross-reference: Level RO SRO Tier # 1 Group # 1 KIA ## 295003 2.1.12 Importance Rating 4.0 Partial or Complete Loss of AC power: Ability to apply Tech Specs for a system.

Proposed Question: SRO 77 Reactor power is 100% with all systems operable and in a normal full power configuration, when 480 VAC motor control center B-15 is lost. Operators stabilize plant conditions by performing the following actions in accordance with station procedures:

0 Cross-tie RBCCW Close SSW Cross-Over Valve MO-3808 Regarding the technical specifications for RBCCW and SSW, which one of the following is required and the bases for that requirement?

A. Be in cold shutdown within 24 hours2.777778e-4 days <br />0.00667 hours <br />3.968254e-5 weeks <br />9.132e-6 months <br /> because both SSW subsystems are inoperable.

B. Be in cold shutdown within 24 hours2.777778e-4 days <br />0.00667 hours <br />3.968254e-5 weeks <br />9.132e-6 months <br /> because both RBCCW subsystems are inoperable.

L/ C. Be in cold shutdown within 24 hours2.777778e-4 days <br />0.00667 hours <br />3.968254e-5 weeks <br />9.132e-6 months <br /> because the Ultimate Heat Sink is inoperable.

D. Restore BOTH the A RBCCW AND ASSW subsystems to operable status within 72 hours8.333333e-4 days <br />0.02 hours <br />1.190476e-4 weeks <br />2.7396e-5 months <br /> due to the loss of associated cooling water pumps.

Proposed Answer: B Explanation (Optional):

A. Incorrect response: The ASSW subsystem is inoperable, but the B subsystem is operable Tech Spec Bases page B3/4.5-12because the SSW cross-over valve was closed.

8. Correct response: Both RBCCW subsystems are inoperable because the cross-connect valves are open (Tech Spec Bases page B3/4.5-12)

C. Incorrect response: The ultimate heat sink remains operable (Tech Spec Bases page B3/4.5-12)

D. Incorrect response: Although both A RBCCW and SSW subsystems are inoperable, the more limiting LCO for both RBCCW subsystems is applicable.

Technical Reference(s) Tech Spec 3-59B3.c (Attach if not previously provided)

Tech Spec 3.5, B.4.B Tech Spec Bases page B3/4.5-12 Page 3 of 51

ES-401 Sample Written Examination Form ES-401-5 Question Worksheet Proposed references to be provided to applicants during examination: TS - no bases L.l Learning Objective: (As available)

Question Source: Bank #

Modified Bank # (Note changes or attach parent)

New X Question History: Last NRC Exam Question Cognitive Level: Memory or Fundamental Knowledge Comprehension or Analysis X 10 CFR Part 55 Content: 55.41 55.43 2 Comments:

Page 4 of 51

ES-401 Sample Written Examination Form ES-401-5 Question Worksheet Examination Outline Cross-reference: Level RO SRO

  • W Tier # 1 Group # 1 WA # 295006 2.1.23 Importance Rating 4.0 Conduct of Operations: Ability to perform specific system and integrated plant procedures during all modes of operation.

Proposed Question: SRO 78 The plant has been scrammed following a small instrument line break in the drywell.

The RPV is being cooled down using the Main Turbine bypass valves. Plant conditions are as follows.

0 RPV Level is + 25 inches and stable.

0 RPV Pressure is 78 psig and decreasing.

0 Drywell Pressure is 2.5 psig and decreasing.

With the conditions stated above, which one of the following actions are required per EOPs to complete the cooldown.

L, A. Secure Torus Spray and place RHR in shutdown cooling in accordance with PNPS 2.2.1 9.1.

B. Continue the cooldown using the main turbine bypass valves and alternate pressure control systems, if necessary, per EOP-01.

C. Defeat the RClC low pressure isolation signal in accordance with PNPS 5.3.21 Bypassing Selected Interlocks, then use RCIC in pressure control to complete the cooldown.

D. Secure Drywell Spray and place RHR in shutdown cooling in accordance with PNPS 2.2.19.1.

Page 5 of 51

ES-401 Sample Written Examination Form ES-401-5 Question Worksheet Proposed Answer: B

'\4 Explanation (Optional):

A Incorrect. RHR cannot be placed in service at until reactor pressure is 75 psig (EOP-01 step P-7). Torus spray in service should not be secured until 0 psig dw pressure.

B Correct. EOP-01, step P use main turbine bypass valves or alternate pressure control systems, if necessary, to continue the cooldown.

A. Incorrect. This is not necessary with the main turbine bypass valves available.

B. Incorrect. Drywell spray should not be in service. There is no indication in stem that containment parameters degraded to that extent. Also, RHR cannot be placed in service until reactor pressure is 75 psig (EOP-01 step P-7).

Technical Reference(s) EOP-01 (Attach if not previously provided)

Proposed references to be provided to applicants during examination: EOPs - no entry conditions Learning Objective: (As available)

Question Source: Bank # EOPS-

  1. 6729 Modified Bank # (Note changes or attach parent)

New Question History: Last NRC Exam Question Cognitive Level: Memory or Fundamental Knowledge Comprehension or Analysis X 10 CFR Part 55 Content: 55.41 55.43 5 Comments:

Page 6 of 51

ES-401 Sample Written Examination Form ES-401-5 Question Worksheet Examination Outline Cross-reference: Level RO SRO i_/

Tier # 1 Group # 1 KIA # 295021 2.4.4 Importance Rating 4.3 Emergency Procedures/Plan ability to recognize abnormal indications for system operating parameters which are entry level conditions for emergency and abnormal operating procedures.

Proposed Question: SRO 79 The reactor was shutdown 48 hours5.555556e-4 days <br />0.0133 hours <br />7.936508e-5 weeks <br />1.8264e-5 months <br /> ago for a refueling outage. Shutdown cooling is then lost and cannot be immediately re-established. Current plant conditions are as follows:

Moderator temperature is 150 degrees and rising 0 Vessel level is + 62 inches 0 Drywell head has been removed 0 Vessel head has been de-tensioned 0 Current time is mid-night (0O:OO)

Utilizing Attachment 5 of procedure 2.4.25, Loss of Shutdown Cooling, and assuming that decay heat removal cannot be re-established, L-The emergency plan should be entered at time:

A. 01:lO AM and an Unusual Event declared

9. 01 :42 AM and an Unusual Event declared C. 01:I 0 AM and an Alert declared D. 01:42 AM and an Alert declared Proposed Answer: C Explanation (Optional): Candidate must recognize that sheet 2 of 2 is applicable, based on current water level and determine heatup rate. Heatup rate 48 hrs after SID is 52.7 (53 degrees). Given initial temperature of 150 degrees, it will take 1.I 6 hours6.944444e-5 days <br />0.00167 hours <br />9.920635e-6 weeks <br />2.283e-6 months <br />, or 70 minutes to reach 21 2 degrees. EAL, 6.2.2.2., inability to establish and maintain SDC and temperature > 21 2 degrees is then exceeded.

,- A. Incorrect. An alert is required. Plausible in that EAL 6.1 .I .I, requires that an Unusual event be declared when the plant cannot be placed in the required

- u Page 7 of 51

ES-401 Sample Written Examination Form ES-401-5 Question Worksheet operating mode in the specified Tech Spec LCO time. The situation provided does require entry into Tech Specs due to loss of primary containment integrity above 212

'4 degrees. .

B. incorrect. An alert is required.

C. Correct Response D. Incorrect: Although an Alert is correct, the time given is based on the time to boil provided in Attachment 2. This time is based on an initial temperature of 120 degrees, which is not correct. The time to boil must be manually calculated.

Technical Reference(s) PNPS 2.4.25 and EPIP- (Attach if not previously provided) 100.1 Proposed references to be provided to applicants during examination: 2.4.25 attachment 5, sheet 2 of 2 Learning Objective: (As available)

Question Source: Bank #

Modified Bank # (Note changes or attach parent)

New X Question History: Last NRC Exam Question Cognitive Level: Memory or Fundamental Knowledge X Comprehension or Analysis 10 CFR Part 55 Content: 55.41 55.43 5 Comments:

Page 8 of 51

ES-401 Sample Written Examination Form ES-401-5 Question Worksheet Examination Outline Cross-reference: Level RO SRO b

Tier # 1 Group # 1 W A ## 295024 EA2.03 Importance Rating 3.8 Ability to determine andlor interpret the following as they apply to high drywell pressure: drywell pressure.

Proposed Question: SRO 80 The following conditions exist following a feedwater line break in the drywell. All equipment is operable.

Reactor is shutdown.

Reactor is depressurized.

Reactor Level is -20 inches and recovering.

Drywell Pressure is 12 psig and rising.

Drywell Temperature is 250 degrees and rising.

Torus Bottom Pressure is 18 psig and slowly rising.

Torus Level is 130 inches and slowly rising Torus Temperature is 150 degrees and rising slowly.

c-Which ONE (1) of the following actions are required per the EOPs to address containment parameters?

A. Place Torus Spray and Drywell Spray in service.

B. Place Torus Spray and Drywell Spray in service and isolate non-essential heat loads in RBCCW.

C. Place ONLY Drywell Spray in service.

D. Emergency RPV Depressurize per EOP-17.

Page 9 of 51

ES-401 Sample Written Examination Form ES-401-5 Question Worksheet Proposed Answer: A L-- Explanation (Optional):

A. Correct. Per EOP-03, primary containment pressure leg, torus spray is required before torus bottom pressure exceeds 16 psig. Drywell spray is required due to rising DW temps.

On the Torus Temperature Leg, with temperature > 130 degrees F. and a LOCA inside primary containment (step TT-7), you are required to isolate non-essential RBCCW heat loads. In this case no LOCA exists with a feed line break. Not required B Incorrect. Not a LOCA, non essential loads should not be isolated.

C. Incorrect. Torus spray also required.

D. Incorrect. No given conditions require Emergency RPV Depressurization.

TechnicaI Reference(s) EOP-03 (Attach if not previously provided)

Proposed references to be provided to applicants during examination: EOPS - no entry conditions Learning Objective: (As available)

Question Source: Bank #

Modified Bank # (Note changes or attach parent)

New X Question History: Last NRC Exam Question Cognitive Level: Memory or Fundamental Knowledge Comprehension or Analysis X 10 CFR Part 55 Content: 55.41 55.43 5 Comments:

Page 10 of 51

ES-401 Sample Written Examination Form ES-401-5 Question Worksheet Examination Outline Cross-reference: Level RO SRO L/

Tier # 1 Group # 1 WA ## 295037 EA2.07 Importance Rating 4.0 Ability to determine and interpret the following as they apply to Scram Conditions Present and RX Power > APRM Downscale or Unknown: Containment Conditions.

Proposed Question: SRO 81 A spurious MSIV closure occurred at 100% reactor power. On the scram signal, a leak developed in the drywell and no rods inserted. Reactor pressure peaked at 1265 psig.

EOP-02 and EOP-03 were entered. The following parameters now exist.

Torus Water Temperature is 160 degrees F. and stable.

Torus Water Level is >300 inches.

Drywell Pressure is 43 psig and slowly rising.

Torus Bottom Pressure is 56 psig and slowly rising.

Only one low pressure ECCS pump is available, the A Core Spray pump Which ONE (1) of the following EOP actions must be taken?

U A. Anticipate Emergency RPV depressurization and Emergency Vent the Drywell, bypassing interlocks and irrespective of adequate core cooling.

6. Perform Emergency RPV Depressurization and Emergency Vent the Drywell irrespective of offsite release.

C. Spray the Torus and Drywell, bypassing interlocks as required to establish the flowpath and Emergency vent the Torus.

D. Spray the Torus and Drywell, bypassing interlocks as required to establish the flowpath and Emergency Vent the Drywell irrespective of offsite release.

Page 11 of 51

ES-401 Sample Written Examination Form ES-401-5 Question Worksheet Proposed Answer: E3 c- Explanation (Optional):

A. Incorrect. You are beyond the point of RPV depressurization B. Correct. The Primary Containment Pressure Limit (PCPL) is being approached quickly(E0P-I 1, figure 4) and per step Of-03, step P-7,emergency vent of the drywell is required. The drywell is vented vice the torus based on EOP-03guide next to step P-7.

C. Incorrect. Drywell must be vented to prevent exceeding the PCPL, D. Incorrect. Only Core Spray is available. EOPs direct emergency drywell Vent, Technical Reference@) EOP step P-7,EOP-11 (Attach if not previously provided)

Figure 4 Proposed references to be provided to applicants during examination: EOPs no entry conditions Learning Objective: (As available)

Question Source: Bank # EOPs

  1. 6662 Modified Bank # (Note changes or attach parent)

New Question History: Last NRC Exam Question Cognitive.Level: Memory or Fundamental Knowledge Comprehension or Analysis X 10 CFR Part 55 Content: 55.41 55.43 5 Comments:

Page 12 of 51

ES-401 Sample Written Examination Form ES-401-5 Question Worksheet Examination Outline Cross-reference: Level RO SRO i/

Tier # 1 Group # 1 KIA # 295028 2. I.20 importance Rating 4.2 High DW temps: Conduct of operations, ability to execute procedure steps.

Proposed Question: SRO 82 A generator load reject has occurred coincident with a loss of offsite power. The current plant conditions presently exist. All other equipment is operable.

0 Reactor is scrammed.

0 Drywell Pressure is 4 psig and slowly rising.

0 Drywell temperature is 195 degrees and steadily rising.

Which ONE (I) of the following steps are you required to direct per plant procedures.

A. Cross tie RBCCW to maximize drywell cooling B. Initiate Drywell Sprays ONLY.

C. Initiate Drywell Sprays, defeat drywell cooler load shed relays and restart the drywell coolers.

D. Defeat drywell cooler load shed relays and restart the drywell coolers.

Proposed Answer: D Explanation (Optional):

A. Incorrect. - Under these conditions not directed by plant procedures.

rr B. Incorrect.

C. Incorrect. "

D. Correct. Per 2.2.19.5. and 2.4.44. - DW coolers have been load shed.

Technical Reference@) PNPS 2.2.1 9.5, PNPS 2.4.44 (Attach if not previously provided)

Page 13 of 51

ES-401 Sample Written Examination Form ES-40 1-5 Question Worksheet Proposed references to be provided to applicants during examination: EOPs - no entry conditions Learning Objective: (As available)

Question Source: Bank # Primary cont.

cooling.

  1. 774 Modified Bank # {Note changes or attach parent)

New Question History: Last NRC Exam Question Cognitive Ll-vel: Memory or Fundamental Knowledge Comprehension or Analysis X 10 CFR Part 55 Content: 55.41 55.43 5 Comments:

Page 14 of 51

ES-401 Sample Written Examination Form ES-401-5 Question Worksheet Examination Outline Cross-reference: Level RO SRO C

Tier # I Group # 1 WA # 295030 EA2.03 Importance Rating 3.9 Ability to determine and interpret the following as they apply to low suppression pool water level: Reactor pressure.

Proposed Question: SRO 83 The plant is in ATVVS conditions following a main steam line break and Group 1 isolation. Plant conditions are as follows.

0 200 gallons of boron were injected before the SLC pumps tripped.

0 Reactor Power is at 30%.

0 Reactor Pressure is 900 psig.

0 Torus Water level is 94 inches and slowly decreasing.

Due to difficulties in establishing suppression pool cooling, torus water temperature cannot be maintained below the HCTL curve.

Which ONE (1) of the following states the correct course of action you must direct to reduce reactor pressure to below the HCTL.

A. Defeat the isolation and reopen the MSIVs.

B. Alternately depressurize with HPCI.

C. Alternately depressurize using RWCU in letdown mode.

D. Emergency RPV depressurization is required.

Page 15 of 51

ES-401 Sample Written Examination Form ES-401-5 Question Worksheet Proposed Answer: D U Explanation (Optional):

A. Incorrect. Not allowed under ATWS conditions. (Override P-4)

B. Incorrect. HPCl can not be used because torus level is below 95 inches , EOP-03, step TL-13 C. Incorrect. Emergency RPV depressurization is required so the use of RWCU is moot.

Also, use of RWCU is not permitted since boron was injected (Table M IEOP-02, step P-8)

D. Correct. Per EOP -03, if torus temps cant be maintained below HCTL, emergency rpv depressurization is required.

Technical Reference(s) EOP-02, RPV pressure leg, (Attach if not previously provided)

EOP-03, Torus Temperature leg Proposed references to be provided to applicants during examination: EOPs - no entry conditions Learning Objective: (As available)

Question Source: Bank # EOPS-

  1. 3350 Modified Bank # (Note changes or attach parent)

New Question History: Last NRC Exam Question Cognitive Level: Memory or Fundamental Knowledge Comprehension or Analysis X 10 CFR Part 55 Content: 55.41 55.43 2, 5 Comments:

Page 16 of 51

ES-40 I Sample Written Examination Form ES-401-5 Question Worksheet Examination Outline Cross-reference: Level RO SRO u

Tier # 1 Group # 2 KIA # 295010 2.4.30 Importance Rating 3.6 Emergency Procedures / Plan knowledge of which events related to systems operations/statusshould be reported to outside agencies.

Proposed Question: SRO 84 Page 17 of 51

ES-401 Sample Written Examination Form ES-401-5 Question Worksheet The plant is operating at 100% power with all systems operable when a scram occurs e due to an apparent large line break in the drywell.

The following conditions now exist.

0 Reactor is shutdown Drywell Pressure is 9 psig and rising Reactor Level is -80 inches and rising Reactor Pressure is 100 psig and lowering Torus Temperature is 105 degrees F.

90 minutes later:

Drywell Pressure is 10 psig and rising 0 Reactor Level is -70 inches and rising 0 Reactor Pressure is 50 psig 0 Torus Temperature is 113 degrees F.

0 Reactor Coolant sample is 220 microcuries.

A. The NRC was required to be notified within one hour due to the high drywell pressure condition. The NRC is also required to be initially notified within one hour of the torus temperature exceeding 110 degrees and the high reactor coolant sample activity.

v B. The NRC was required to be notified within 15 minutes due to the high drywell pressure condition. The NRC is also required to be initially notified within 15 minutes of torus temperature exceeding 110 degrees and the high reactor coolant sample activity.

C. The NRC was required to be notified within one hour due to the high drywell pressure condition. A separate initial notification to the NRC is NOT required within one hour of the torus temperature exceeding I10 degrees and the high reactor coolant sample activity.

D. The NRC was required to be notified within 15 minutes due to the high drywell pressure condition. A separate initial notification to the NRC is NOT required within one hour of the torus temperature exceeding 110 degrees and the high reactor coolant sample activity.

Page I 8 of 51

ES-401 Sample Written Examination Form ES-401-5 Question Worksheet Proposed Answer: C ii' Explanation (Optional):

A. Incorrect. Separate notification not required per EP-IP-I00.

B. Incorrect. One hour reports per EP-IP-100.

C. Correct. The initial must be reported within the one hour requirements per 10 CFR 50.72 -

Drywell Press > 2.2 psig alert (3.4.1.2). Torus Temp > 110 -UE. Reactor Coolant sample

.I .2) but does not need to be independently reported.

activity is also an Alert (1.I D. Incorrect. One hour not 15 minutes.

Technical Reference(s) EP-IP-I00.1, Rev.3, (Attach if not previously provided)

Emergency action levels, Section 3.4.1.2for DW press >

2.2 psig (alert), Section 3.2.1.1.

for the torus temp >I10 (UE)

Proposed references to be provided to applicants during examination: EAL Chart Learning Objective: (As available)

Question Source: Bank #

Modified Bank # (Note changes or attach parent)

New X Question History: Last NRC Exam Question Cognitive Level: Memory or Fundamental Knowledge Comprehension or Analysis X 10 CFR Part 55 Content: 55.41 55.43 2, 5 Comments:

Page 19 of 51

ES-401 Sample Written Examination Form ES-401-5 Question Worksheet e Examination Outline Cross-reference: Level RO SRO Tier # 1 Group # 2 WA # 295010 AA2.01 Importance Rating 3.9 Ability to determine andlor interpret the following as they apply to high drywell temperature: Drywell temperature.

Proposed Question: SRO 85 Following a major LOCA you are executing EOPs and need to determine drywell bulk temperature. Containment conditions are significantly degraded.

Which ONE (I) of the following methods for determining drywell bulk temperature is environmentally qualified to be used in a harsh environment under accident conditions?

A. An average of the TE-5050 series RTDs using EPIC computer points.

B. An average of the TE-8125 series RTDs using the KAYE computer.

C. The higher value of TI-9019 or TRU 9044 Drywell Temperature Indicator and u

Recorder on the C903 panel.

D. Locally from the TE-5050E series temperature elements on Panel C85 in the Reactor Building.

Proposed Answer: C Explanation (Optional):

A. Incorrect. Not environmentally qualified for a harsh environment per PNPS 2.1.27.

B. Incorrect. Not environmentally qualified for a harsh environment per PNPS 2.1.27.

C. Correct. Per PNPS 2.1.27 this is the only method that is environmentally qualified for a harsh environment.

D. Incorrect. Not environmentally qualified for a harsh environment per PNPS 2.127.

Technical Reference(s) PNPS 2.1.27, Rev. 7, Drywell (Attach if not previously provided) temperature Indication step 7.0[1J Page 20 of 51

ES-401 Sample Written Examination Form ES-401-5 Question Worksheet Proposed references to be provided to applicants during examination: NONE Learning Objective: (As available)

Question Source: Bank #

Modified Bank # (Note changes or attach parent)

New X Question History: Last NRC Exam Question Cognitive Level: Memory or Fundamental Knowledge Comprehension or Analysis X 10 CFR Part 55 Content: 55.41 55.43 5 Comments:

Page 21 of 51

ES-401 Sample Written Examination Form ES-401-5 Question Worksheet Examination Outline Cross-reference: Level RO SRO u

Tier # 1 Group # 2 KIA # 295029 2.2.25 Importance Rating 3.7 Equipment control knowledge of bases in TS for LCOs and safety limits.

Proposed Question: SRO 86 The plant has experienced a LOCA and the following conditions exist 0 Reactor power 0%.

0 Reactor Pressure - 150 psig.

0 Reactor Level (-1 00) slowly lowering.

0 Torus Temperature - 130 degrees F. rising slowly 0 Torus Level - 170 inches rising rapidly Torus Bottom Pressure - 12.5 psig.

- 0 Torus Spray is in service.

Drywell Temperature - 160 degrees F. rising 0 Drywell Pressure is 6.0 psig 0 Drywell Spray is in service Which ONE (I) of the following actions are you now required to direct in accordance with EOPs and why must that action be taken?

A. Secure Drywell Spray per EOP-03 to prevent potentially exceeding the pressure suppression limit.

B. Emergency Depressurize per EOP-01 to prevent uncovering fuel.

C. Secure Torus Spray per EOP-03 to prevent potentially exceeding the primary containment pressure limit.

D. Emergency Depressurize per EOP-03 to prevent potentially exceeding the pressure suppression limit.

Page 22 of 51

ES-401 Sample Written Examination Form ES-401-5 Question Worksheet W

Proposed Answer: D Explanation (Optional):

A. Incorrect. For conditions given.

6. Incorrect. For conditions given.

C. Incorrect. For conditions given.

D. Correct. With rapidly increasing torus level EOP-03 directs at step TL-7 "when torus level cannot be maintained below 175 inches continue to next step which is ED Technical Reference(s) EOP-03, EOP-03 bases (Attach if not previously provided)

Proposed references to be provided to applicants during examination: EOPs - no bases or entry conditions Learning Objective: (As available)

Question Source: Bank #

Modified Bank # (Note changes or attach parent)

New X Question History: Last NRC Exam Question Cognitive Level: Memory or Fundamental Knowledge Comprehension or Analysis X 10 CFR Part 55 Content: 55.41 55.43 2 Comments:

Page 23 of 51

ES-401 Sample Written Examination Form ES-401-5 Question Worksheet Examination Outline Cross-reference: Level RO SRO u

Tier # 1 Group # 2 WA # 500000 EA2.03 Importance Rating 3.8 Ability to determine andlor interpret the following as they apply to High Primary Containment H2 Conc.: Combustible DW limits.

Proposed Question: SRO 87 Severe Accident Conditions are present in containment, Which ONE (1) of the following describes the deflagration limits of Oxygen and Hydrogen concentrations in containment in accordance with PNPS 5.4.6 Primary Containment Venting and Purging Under Emergency Conditions?

A. When 0 2 concentration in the Torus is greater than its deflagration limit of 4%

AND H2 concentration is greater than its deflagration limit of 5%. -

6. When 0 2 concentration in the Torus is greater than its deflagration limit of 5%

AND H2 concentration is greater than its deflagration limit of 6%.

C. When 0 2 concentration in the Drywell is greater than its deflagration limit of 4%

OR H2 concentration is greater than its deflagration limit of 5%.

D. When 0 2 concentration in the Drywell is greater than its deflagration limit of 5%

OR H2 concentration is greater than its deflagration limit of 6%.

Proposed Answer: B Explanation (Optional): SRO only because these are severe accident conditions.

A. Incorrect. Wrong limits

6. Correct.

C. incorrect. Wrong limits.

D. Incorrect. AND not OR.

Technical Reference(s) PNPS 5.4.6, Rev. 39 (Attach if not previously provided) l.J Proposed references to be provided to applicants during examination: NONE Page 24 of 51

ES-401 Sample Written Examination Form ES-401-5 Question Worksheet Learning Objective: (As available)

Question Source: Bank #

Modified Bank # (Note changes or attach parent)

New X Question History: Last NRC Exam Question Cognitive Level: Memory or Fundamental Knowledge X Comprehension or Analysis 10 CFR Part 55 Content: 55.41 55.43 5 Comments:

Page 25 of 51

ES-401 Sample Written Examination Form ES-401-5 Question Worksheet Examination Outline Cross-reference: Level RO SRO U

Tier # 2 Group # 1 WA # 215003 A2.06 importance Rating 3.2 Ability to (a) predict the impacts of the following on the Intermediate Range Monitor System and (b) based on those predictions, use procedures to correct, control, or mitigate the consequencesof those abnormal conditions or operations: Faulty Range Switch :

Proposed Question: SRO 88 A Reactor startup is in progress. IRM B previously failed upscale and it was bypassed.

Repair parts wont be available for 36 hours4.166667e-4 days <br />0.01 hours <br />5.952381e-5 weeks <br />1.3698e-5 months <br />. All other IRMs are operable. Shortly after reaching the point of adding heat the IRM F range switch experiences mechanical binding and cannot be physically moved from its current position. I&C is notified and tells you that the IRM F cant be repaired for 24 hours2.777778e-4 days <br />0.00667 hours <br />3.968254e-5 weeks <br />9.132e-6 months <br />. As the CRS you declare IRM F inoperable.

( I )What Technical Specification action(s) are you required to take in regard to the IRM F inoperability? AND (2) How does Technical Specifications affect the startup?

>uA. (I) Bypass IRM F.

(2) The startup can then continue.

6. (1) Insert a % scram on the Bside of RPS.

(2) Initiate a plant shutdown within 2 hours2.314815e-5 days <br />5.555556e-4 hours <br />3.306878e-6 weeks <br />7.61e-7 months <br /> C. (1) Insert control rods until power is reduced to within SRM range.

(2) The startup can then be resumed once the IRM is repaired and declared operable D. (1) Insert a !4 scram on the B side of RPS.

(2) The startup can then continue until a rod block occurs.

Page 26 of 51

ES-401 Sample Written Examination Form ES-401-5 Question Worksheet Proposed Answer: D L' Explanation (Optional):

A. Incorrect. There is only one IRM physically permitted to be bypassed on any one side of RPS. IRM B is already bypassed.

B. incorrect. The startup may continue with all other IRMs operable.

C. Incorrect. With the 2"dIRM hop on one side of RPS a % scram must be inserted per TS.

D. Correct. With the 2"' IRM inop on one side of RPS a % scram must be inserted per TS then the startup may continue until a rod block occurs as power increases on the 'F' IRM which is stuck in its current range.

Technical Reference(s) TS 3.1.1, Table 3.1.I, action A. (Attach if not previously provided)

Proposed references to be provided to applicants during examination: TS (not bases)

Learning Objective: (As available)

Question Source: Bank #

Modified Bank # PNPS #I63 {Note changes or attach parent)

New Question History: Last NRC Exam Question Cognitive Level: Memory or Fundamental Knowledge Comprehension or Analysis X 10 CFR Part 55 Content: 55.41 2, 5 55.43 Comments:

12/1 - Gil- reformat answers 12/19 -Steve - revised Page 27 of 51

ES-401 Sample Written Examination Form ES-401-5 Question Worksheet Examination Outline Cross-reference: Level RO SRO W

Tier # 2 Group # I KIA # 21 5004 2.2.25 Importance Rating 3.7 Equipment control knowledge of bases in Tech S p e c s for LCOs and safety limits.

Proposed Question: SRO 89 During refueling the plant is performing a core spiral unloading program. The Reactor Operator monitoring the SRMs reports the following:

0 SRM A- reading 4 cps 0 SRM B - reading 2 cps 0 SRM C - reading 3 cps 0 SRM D - reading 3 cps In accordance with technical specification requirements you direct the refueling floor SROto (1) because-(2)  :

4 A. (1) Stop the core spiral unloading (2) Lower than required SRM readings will NOT ensure protection against a rod drop accident.

B. (1) Continue the core spiral unloading.

(2) SRM readings are expected to drop below 3 cps during spiral unloading.

C. (1) Stop the core spiral unloading.

(2) SRM readings will ensure protection against an inadvertent control rod withdrawal.

D. (1) Continue the core spiral unloading.

(2) Lower than required SRM readings will NOT ensure protection against a rod drop accident.

Page 28 of 51

ES-401 Sample Written Examination Form ES-401-5 Question Worksheet Proposed Answer: B Explanation (Optional):

A. Incorrect. Per TS 3.10.b.

B. Correct. Per TS 3.10.b.3 and bases 3.10.b.

C. Incorrect. Per TS and bases 3.10.b.

D. Incorrect. Per TS and bases 3.10.b.

Technical Reference(s) TS and bases section 3.1O.B. (Attach if not previously provided)

Proposed references to be provided to applicants during examination: TS no bases Learning Objective: (As available)

Question Source: Bank #

Modified Bank # (Note changes or attach parent)

New X Question History: Last NRC Exam Question Cognitive Level: Memory or Fundamental Knowledge X Comprehension or Analysis 10 CFR Part 55 Content: 55.41 55.43 2, 6, 7 Comments:

Page 29 of 51

ES-401 Sample Written Examination Form ES-401-5 Question Worksheet Examination Outline Cross-reference: Level RO SRO Tier # 2 Group # 1 KIA # 261000A2.03 Importance Rating 3.2 Ability to predict the impacts of the following on SBGT and based on those predictions use procedures to correct, control, or mitigate the consequences: High train temp.

Proposed Question: SRO 90 Both trains of the SBGT system are in service for a containment vent following a LOCA.

The control room operators observe a control room alarm for High Temperature in SBGT train A.

Local observation shows evidence that a fire had occurred but none is present at this time. Local temperature indication shows Train A temperature at 210 degrees F. Train B temperature is in its normal range.

In accordance with PNPS 2.2.50 Standby Gas Treatment your direction for the operators is to .

A. Ensure that the Train Afan has tripped, close its associated inlet and outlet cross connect valves, and initiate its deluge system. Train B can then remain running.

B. Ensure that the Train A fan has tripped, close its associated inlet and outlet cross connect valves. Train B can then remain running.

C. Secure both trains and initiate their deluge systems.

D. Secure Train A, Train B can remain running unless there are indications of increasing train temperature.

Page 30 of 51

ES-401 Sample Written Examination Form ES-401-5 Question Worksheet Proposed Answer: B L-; Explanation (Optional):

A. Incorrect. The deluge system shall not be operated post accident.

3. Correct. If the cross connect valves are closed to isolate train A, Train B can remain running. Train A fan should have tripped on high train temp (>200 degrees).

C. Incorrect. The deluge system shall not be operated post accident.

D. Incorrect. The cross connect valves must be closed to allow continued operation of Train 6.

Technical Reference@) PNPS 2.2.50, Rev.61 , step (Attach if not previously provided) 5.0141 and step 4.3[3]

Proposed references to be provided to applicants during examination: NONE Learning Objective: (As available)

Question Source: Bank #

Modified Bank # (Note changes or attach parent)

New X Question History: Last NRC Exam Question Cognitive Level: Memory or Fundamental Knowledge Comprehension or Analysis X 10 CFR Part 55 Content: 55.41 55.43 4, 5 Comments:

Page 31 of 51

ES-401 Sample Written Examination Form ES-401-5 Question Worksheet Examination Outline Cross-reference: Level RO SRO 4

Tier # 2 Group # 1 KIA # 262001 2.1.12 Importance Rating 4.0 Conduct of operations: ability to apply TS for a system - ac distribution.

Proposed Question: SRO 91 The plant is operating at 100% power with all equipment operable with the exception of the following.

0 The Shutdown Transformer is out of service for maintenance. It will available in 4 days.

0 AEDG is inoperable due to emergent maintenance and will be available in 24 hours2.777778e-4 days <br />0.00667 hours <br />3.968254e-5 weeks <br />9.132e-6 months <br />.

You then receive a report from the field operator that the B SLC pump has a crack in the pump casing and oil has leaked out onto the floor. Maintenance advises that the pump must be tagged out of service and wont be available for at least 36 hours4.166667e-4 days <br />0.01 hours <br />5.952381e-5 weeks <br />1.3698e-5 months <br />.

L/ The most limiting Technical Specification LCO you are now required to enter is:

A. Continued reactor operation is permitted for 48 hours5.555556e-4 days <br />0.0133 hours <br />7.936508e-5 weeks <br />1.8264e-5 months <br />.

B. Continued reactor operation is permitted for 7 days.

C. Be in Hot Shutdown within the next 20 hours2.314815e-4 days <br />0.00556 hours <br />3.306878e-5 weeks <br />7.61e-6 months <br />.

D. Be in Hot Shutdown within 8 hours9.259259e-5 days <br />0.00222 hours <br />1.322751e-5 weeks <br />3.044e-6 months <br />.

Page 32 of 51

ES-401 Sample Written Examination Form ES-401-5 Question Worksheet Proposed Answer: C 4 Explanation (Optional):

A. Incorrect. Not most limiting - for shutdown transformer and one EDG unavailable with startup transformer available only. (TS 3.9.B.4).

B. Incorrect. Not most limiting - for shutdown transformer inop but all EDGs available (TS 3.9.B.I .b.).

C. Correct. With the shutdown transformer and A EDG inop, TS 3.9.B.4 states that reactor operation for 48 hours5.555556e-4 days <br />0.0133 hours <br />7.936508e-5 weeks <br />1.8264e-5 months <br /> is permitted. TS 3.9.6.3states that with one EDG inop continued operation is permitted if additionally listed TS are met. One of those listed is 3.4.B.1 for SLC operability. With power unavailable to EDG A for the A SLC pump and SLC pump Bwith a cracked casing, both must be considered inoperable. Because no equipment will return within 8 hrs (TS 3.4.B.2) then TS 3.4.D applies to be in Hot Shutdown within the next 12 hours1.388889e-4 days <br />0.00333 hours <br />1.984127e-5 weeks <br />4.566e-6 months <br /> (8 + I 2 = 20).

D. Incorrect. This time limit is concerned only with returning one SLC subsystem to operable status (TS 3.4.D).

Technical Reference(s) TS 3.4 and 3.9 (Attach if not previously provided)

Proposed references to be provided to applicants during examination: TS no bases Learning Objective: (As available)

Question Source: Bank #

Modified Bank # (Note changes or attach parent)

New X Question History: Last NRC Exam Question Cognitive Level: Memory or Fundamental Knowledge Comprehension or Analysis X 10 CFR Part 55 Content: 55.41 55.43 2 Comments:

Page 33 of 51

ES-401 Sample Written Examination Form ES-401-5 Question Worksheet Examination Outline Cross-reference: Level RO SRO LJ Tier # 2 Group # 2 WA # 286000 2.1.23 Importance Rating 4.0 Ability to perform integrated and specific plant procedures during all modes of plant operations.

Proposed Question: SRO 93 The plant is at 75% power when a fire occurs in the cable spreading room. The Fire Brigade reports that it appears that electrical wiring was affected. In accordance with PNPS 2.4.143, Shutdown from Outside the Control Room, equipment control is required to be shifted to -(I)- Alternate Shutdown Panels (ASPs) -(2)-.

A. (1)Ail.

(2) If the control room remains manned.

B. (I) All.

(2) As soon as possible.

C. (1) Only to the HPCl and SRV ASPs.

-- (2) If the control room remains manned.

D. (1) Only to the HPCl and SRV ASPs.

(2) As soon as possible.

Proposed Answer: B Explanation (0ptional):

A. Incorrect. Not in accordance with abnormal B. Correct. PNPS 2.4.143, Shutdown from Outside the Control Room, Note on page 9, that states, in part, It is imperative to perform steps [I61 and (171 as soon as possible to prevent spurious operation of equipment in the evnt of a CSR or MCR fire. Below is Note 16, Note 17 refers to breakers

[IS] E a fire or damage in the CSR or MCR that destroys electrical wiring has occurred, THEN, at each of t h e following Alternate Shutdown Panels (ASPS),TRANSFER contrul of included components ts the ASP ED accordance with Section 3.1of the appropriate Appendix:

See procedure for complete list of ASPs (RCIC, RHR, EDGs also included).

Page 36 of 51

ES-401 Sample Written Examination Form ES-401-5 Question Worksheet C. Incorrect. Not in accordance with abnormal D. Incorrect. Not in accordance with abnormal Technical Reference(s) PNPS 2.4.143.2 - Shutdown (Attach if not previously provided) with a fire in Reactor Building West. Rev.15, Note on page 9

Proposed references to be provided to applicants during examination: NONE Learning Objective: (As available)

Question Source: Bank # Fire Protection

  1. 6057 Modified Bank # (Note changes or attach parent)

New Question History: Last NRC Exam Question Cognitive Level: Memory or Fundamental Knowledge X Comprehension or Analysis 10 CFR Part 55 Content: 55.41 55.43 5 Comments:

Page 37 of 51

I . .

ES-401 Sample Written Examination Form ES-401-5 Question Worksheet Examination Outline Cross-reference: Level RO SRO d

Tier # 3 Group #

WA # G2.1 .I I

importance Rating 3.8 Knowiedge of less than one hour technical specification action statements for systems.

Proposed Question: SRO 94 While operating at power annunciator 905L-B6, RPS A TESTIFAILURE alarms. The turbine building operator reports that LIS-263-57A (Reactor Low-Low Water Level -46) indicates upscale, its gross failure light is illuminated and will not reset. I&C reports that LIS-263-57A will take approximately five hours to repair.

Assume all other equipment is operable The required action is to initiate an LCO and  ?

A. Direct I&C to trip the Group I PClS Atrip system within one (1) hour.

4 B. Direct I&C to trip the Group I PClS Atrip system within two (2) hours.

C. If I&C has not repaired LIS-263-57A within six (6) hours, direct I&C to trip the Group I PClS A trip system.

D. If i&C has not repaired LIS-263-57A within twelve (12) hours, direct I&C to trip the Group I PClS A trip system.

Proposed Answer: A Explanation (Optional):

A. Correct. Per TS table 3.2.A Notes #2 Action second paragraph.

B. incorrect. Per TS table 3.2.A requirements.

C. Incorrect. Per TS table 3.2.A requirements.

D. incorrect. Per TS table 3.2.A requirements.

Technical Reference(s) TS table 3.2.A. Notes #2 action (Attach if not previously provided)

Page 38 of 51

ES-401 Sample Written Examination Form ES-401-5 Question Worksheet Proposed references to be provided to applicants during examination: Provide vessel L

instrumentation print (M-253, sht 1.) and TS section 3 Learning Objective: (As available)

Question Source: Bank # PNPS #804 Modified Bank # (Note changes or attach parent)

New Question History: Last NRC Exam Question Cognitive Level: Memory or Fundamental Knowledge Comprehension or Analysis X 10 CFR Part 55 Content: 55.41 55.43 5 Comments:

12/1 - Gil - add assume all other equipment operable 12/19 - revised Page 39 of 51

ES-401 Sample Written Examination Form ES-401-5 Question Worksheet Examination 0utline Cross-reference: Level RO SRO Tier # 3 Group #

WA # G2.2.25 Importance Rating 3.7 Knowledge of bases in technical specifications for limiting conditions for operations and safety limits.

Proposed Question: SRO 95 The plant has been operating at 100% power, steady state, for 533 days. The following information is on the most current 3-D Monicore log.

MFLCPR = 1.01 MFLPD = 1.03 MAPRAT = 1.07 PCRAT = 1.02 What mode of fuel damage, if any, is taking place at this time?

A. There is no fuel damage taking place.

B. The fuel is presently >2000 degrees F.

C. The fuel has experienced departure from nucleate boiling.

D. The fuel is experiencing plastic strain >I %.

Proposed Answer: D Explanation (Optional):

A. Incorrect.

6. Incorrect.

C. Incorrect.

D. Correct.

Technical Reference(s) FSAR or current reload (Attach if not previously provided) analysis. NEED REF.

Proposed references to be provided to applicants during examination: NONE Learning 0bjective: (As available)

Page 40 of 51

ES-401 Sample Written Examination Form ES-401-5 Question Worksheet Question Source: Bank # PNPS

  1. 6076 Modified Bank # (Note changes or attach parent)

New Question History: Last NRC Exam Question Cognitive Level: Memory or Fundamental Knowledge X Comprehension or Analysis 10 CFR Part 55 Content: 55.41 55.43 1, 2 Comments:

12/1 - Gil - no comments Page 41 of 51

ES-401 Sample Written Examination Form ES-401-5 Question Worksheet Examination Outline Cross-reference: Level RO SRO

-d Tier# . 3 Group #

KIA # 2.2.32 Importance Rating 3.3 Knowledge of the effects of alterations on core configuration.

Proposed Question: SRO 96 Given the following conditions:

0 You are the Refueling SRO with the plant in a refueling outage and a fuel shuffle in progress.

0 Fuel is currently being removed around control rod 38-07.

SRM B is currently inoperable SRMs AC, and D are functioning properly.

0 I&C would like to tagout and remove one of the other SRM detectors.

NOTE: Refer to the attached Core Maps.

Under these conditions, in accordance with technical specifications, a removal of the 4

-(1)- SRM detector -(2)- be permitted.

A. (I) A (2) would not B. (1)C (2)would C. (I) D (2)would D. (I) D (2) would not Proposed Answer: D Explanation (Optional):

A. Incorrect. No SRMs in rod 38-07 adjacent quadrant.

B. Incorrect. Would be allowed.

C. Incorrect. Would not be allowed - no operable SRM in quadrant D. Correct. No operable SRM in quadrant of where fuel is being moved (rod 30-07).

Page 42 of 51

ES-401 Sample Written Examination Form ES-401-5 Question Worksheet Technical Reference(s) TS definitions, TS 3.10.B (Attach if not previously provided)

'u' PNPS 4.3 Rev 107 Pgs. 49-52 Proposed references to be provided to applicants during examination: TS (not bases or definitions). Core Map Learning Objective: (As available)

Question Source: Bank #

Modified Bank # X (Note changes or attach parent)

New Question History: Last NRC Exam 8/2002 Question Cognitive Level: Memory or Fundamental Knowledge Comprehension or Analysis X 10 CFR Part 55 Content: 55.41 55.43 6 Comments:

12/1- Gil -too easy

-- 12/19- Steve - revised Page 43 of 51

ES-401 Sample Written Examination Form ES-401-5 Question Worksheet Examination Outline Cross-reference: Level RO SRO w'

Tier # 3 Group #

WA # 2.3.4 Importance Rating 3.1 Knowledge of radiation exposure limits and contaminationcontrol, including permissible levels in excess of those authorized.

Proposed Question: SRO 97 During an emergency, a task requires an extended dose limit.

If the task exposure exceeds-(l)- Rem an extended dose limit must be determined and authorized by the -(2)-.

A. (1) 4 (2) Emergency Director OR Emergency Plant Manager for onsite personnel when delegated.

6. (1) 4 (2) Emergency Director ONLY.

C.

(2) Emergency Director OR Emergency Pldnt lanager for onsite personnel when delegated.

D. (1) 5 (2) Emergency Director ONLY.

Proposed Answer: C Explanation (Optional):

A. Incorrect. Limit 5 rem.

B. Incorrect. Limit is 5 rem, the EPM can also authorize.

C. Correct. 5 rem, ED and EPM - per EPIP step 6.3[5]

D. Incorrect.

Technical Reference(s) EP IP-44O,Rev.7 step 6.3[5] (Attach if not previously provided)

Page 44 of 51

ES-401 Sample Written Examination Form ES-401-5 Question Worksheet Proposed references to be provided to applicants during examination: NONE Learning Objective: (As available)

Question Source: Bank #

Modified Bank # (Note changes or attach parent)

New X Question History: Last NRC Exam Question Cognitive Level: Memory or Fundamental Knowledge X Comprehension or Analysis I O CFR Part 55 Content: 55.41 55.43 4 Comments:

Page 45 of 51

ES-401 Sample Written Examination Form ES-401-5 Question Worksheet W

Examination Outline Cross-reference: Level RO SRO Tier # 3 Group #

WA # 2.3.9 Importance Rating 3.4 Knowledge of the process for performing a containment purge.

Proposed Question: SRO 98 You are the Control Room Supervisor. You are performing a drywell purge. One step that may be performed as part of the purge process requires specific approval prior to being performed.

(1) Which one of the following actions requires specific approval prior to being performed.

(2) By position title, who must you obtain specific approval from prior to directing the action.

A. (I ) Opening the Primary Containment to Contaminated Exhaust Damper AO-N-139.

(2) The Shift Manager.

B. (I) Opening the Primary Containment to Contaminated Exhaust Damper AO-N-139.

(2) The Operations Manager.

C. (I) Placing the control switch for the Torus 2vent valves to the EMERGENCY OPEN position.

(2) The Shift Manager.

D. (1) Placing the control switch for the Torus 2 vent valves to the EMERGENCY OPEN position.

(2) The Operations Manager.

Page 46 of 51

ES-401 Sample Written Examination Form ES-401-5 Question Worksheet Proposed Answer: C Explanation (Optional):

A. Incorrect. Wrong action.

B. Incorrect. Wrong approver.

C. Correct. Per PNPS 2.2.70, Rev.98 Primary Containment Atmospheric Control System.

Att. 12 , step 2.0[4]

D. Incorrect. Wrong approver.

Technical Reference(s) PNPS 2.2.70,Rev.98 Primary (Attach if not previously provided)

Containment Atmospheric Control System. Att. 12,step 2.0 [4]

Proposed references to be provided to applicants during examination: NONE Learning Objective: (As available)

Question Source: Bank #

Modified Bank # (Note changes or attach parent)

New X L-Question History: Last NRC Exam Question Cognitive Level: Memory or Fundamental Knowledge X Comprehension or Analysis 10 CFR Part 55 Content: 55.41 55.43 5 Comments:

Page 47 of 51

ES-401 Sample Written Examination Form ES-401-5 Question Worksheet 4 Examination Outline Cross-reference: Level RO SRO Tier # 3 Group #

KIA # 2.4.41 Importance Rating 4.3 Knowledge of the parameters and logic used to assess the status of safety functions including: 1 reactivity Control 2. Core Cooling and heat removal. 3. Reactor coolant system integrity 4. Containmentconditions. 5. Radioactivity release control.

Proposed Question: SRO 99 A primary system is currently discharging into secondary containment.

Which ONE of the following requires entry into EOP-17?

A. RHR 6 and D pump area stairwell is I 9 0 degrees F. AND RHR A and C pump area is 230 degrees F.

B. BRHR quadrant water level is at 8 inches above floor elevation AND Reactor Building West area radiation level is at 1100 mr/hr.

L 4 C. HPCl turbine area temperature is at 180 degrees F. AND RHR 6 and D pump area stairwell is at 205 degrees F.

D. B RHR quadrant water level is at 8 inches above floor elevation AND HPCl Turbine Area is at 170 degrees F.

Proposed Answer: C Explanation (Optional):

A. Incorrect. Parameters dont exceed Max Safe Operating limits in one area.

B. Incorrect. Parameters dont exceed Max Safe Operating limits in one area.

C. Correct. These 2 parameters exceed the Max Safe Operating Values for one area in EOP-04, Table L. With a primary system discharging into secondary containment, Per EOP-04, step sc-16, Emergency RPV Depressurizationis required (EOP-17).

D. Incorrect. Parameters dont exceed Max Safe Operating limits in one area.

Technical Reference(s) EOP-04 (Attach if not previously provided)

~

Page 48 of 51

ES-401 Sample Written Examination Form ES-401-5 Question Worksheet Proposed references to be provided to applicants during examination: EOPs - no entry conditions Learning Objective: (As available)

Question Source: Bank # EOPs

  1. 3319 Modified Bank # (Note changes or attach parent)

New Question History: Last NRC Exam Question Cognitive Level: Memory or Fundamental Knowledge Comprehension or Analysis X 10 CFR Part 55 Content: 55.41 55.43 5 Comments:

4 Page 49 of 51

ES-401 Sample Written Examination Form ES-401-5 Question Worksheet be closed for the reason provided if there is evidence of a RWCU high area ternperatu re.

D. Incorrect response. These valves are part of the IST program.

Technical Reference(s) ARP 904LGA6, PNPS (Attach if not previously provided) 2.4.40, page 6, 2"'

paragraph.

Proposed references to be provided to applicants during examination: NONE Learning Objective: (As available)

Question Source: Bank #

Modified Bank # (Note changes or attach parent)

New X Question History: Last NRC Exam Question Cognitive Level: Memory or Fundamental Knowledge X Comprehension or Analysis 10 CFR Part 55 Content: 55.41 55.43 5 Comments:

12/28 - Gil -KA eval - actions directed from procedure guidance 1/8 - Steve - new question chosen.

Page 51 of 51